EXAM 7 Hematology Oncolgy

Ace your homework & exams now with Quizwiz!

Glucose is important to the body because?

The body naturally tightly regulates blood glucose levels as a part of metabolic homeostasis. Glucose is the primary source of energy for the body's cells Glucose is transported from the intestines or liver to body cells via the bloodstream, and is made available for cell absorption via the hormone insulin, produced by the body primarily in the pancreas.

What is the white layer that is between the RBC's and the Plasma layer? What does it contain?

The buffy coat, WBC's, platelets, +/- microfilariae, +/- NRBC's. Nucleated RBCs (NRBCs) are immature RBCs are not normally seen in the peripheral blood beyond the neonatal period.

"A nurse is caring for a 25-year-old client with end-stage testicular cancer who has been referred to hospice care. Which of the following criteria excludes the client from hospice care?

The client entered a clinical trial through the National Cancer Institute

To treat cervical cancer, a client has had an applicator of radioactive material placed in the vagina. Which observation by the nurse indicates a radiation hazard?

The client receives a complete bed bath each morning.

A nurse is collecting a health history from a client. Which of the following findings is the highest risk factor for the client developing bladder cancer? The client is a hairdresser. The client uses tobacco. The client is over 60 years of age. The client has frequent urinary tract infections (UTIs)

The client uses tobacco. · The nurse should apply the safety and risk reduction priority-setting framework. This framework assigns priority to the factor or situation posing the greatest safety risk to the client. When there are several risks to client safety, the one posing the greatest threat is the highest priority. The nurse should use Maslow's hierarchy of needs, the ABC priority-setting framework, or nursing knowledge to identify which risk poses the greatest threat to the client. Therefore, the nurse should identify the client's tobacco use as being the greatest risk factor for developing bladder cancer.

The nurse is preparing a client for magnetic resonance imaging (MRI) to confirm or rule out a spinal cord lesion. During the MRI scan, which of the following would pose a threat to the client?

The client wears a watch and wedding band

FALSE

The correct term for an increase in neutrophils is neutrocytosis

Cytoplasm is

The fluid portion of a cell, excluding the nucleus

Plasma is....

The fluid portion of anti-coagulated whole blood - would come from a LTT, Gray TT, or Green TT. Plasma has two acceptable "normal" color options: Clear or Straw.

Serum is....

The fluid portion of coagulated blood - would come from RTT, Serum Seperator Tube (Marble Top Tube),

Chromatin is

The genetic material of the cell nucleus

MCHC (Mean Corpuscular Hemoglobin Concentration) is..... Formula for MCHC calculation is: Hb (hemoglobin) X 100 / (divided by) PCV (packed cell volume) This is rounded to tenths place and reported in g/dl Do cells have the correct amount of Hb (hemoglobin) in them for that species?

The mean corpuscular hemoglobin concentration, a measure of the concentration of hemoglobin in a given volume of packed red blood cells. It is reported as part of a standard complete blood count. Reported as NR = Normochromic Higher than normal = Hyperchromic Lower than normal = Hypochromic (cells are lacking in hemoglobin (Hb)

MCV (Mean Cell Volume) is... Formula for MCV=PCV(packed cell volume) X 10 /(divided by) RBC's = (fl) fentoliters is unit - round to tenths place Microcytic cells are smaller than they should be for a species Macrocytic cells are larger than they should be for a species

The mean corpuscular volume, or "mean cell volume" (MCV), is a measure of the average red blood cell size that is reported as part of a standard complete blood count. The MCV is calculated by dividing the total volume of packed red blood cells (also known as hematocrit) by the total number of red blood cells. The resulting number is then multiplied by 10. The red blood cells get packed together when they are spun around at high speeds in a centrifuge. In patients with anemia, it is the MCV measurement that allows classification as either a microcytic anemia (MCV below normal range), normocytic anemia (MCV within normal range) or macrocytic anemia (MCV above normal range).

FALSE

The order of the granulocyte pools is: Dividing, Maturation, Storage, Marginating, Circulating & the tissues

The retic count is used for?

The reticulocyte count is used to help determine if the bone marrow is responding adequately to the body's need for red blood cells (RBCs) and to help determine the cause of and classify different types of anemia. The number of reticulocytes must be compared to the number of RBCs to calculate a percentage of reticulocytes; so the test is ordered along with a RBC count. A hemoglobin and/or hematocrit are usually ordered in order to evaluate the severity of anemia.

Thrombocytes are...

The smallest formed elements in the blood. There purpose is blood clotting and they are formed in the red bone marrow.

FALSE

The stimulus for production of basophil is allergens and they produce antihistamine

Mycoplasmas are different from other bacteria because they do not have a cell wall surrounding and protecting their microscopic bodies.

They cannot be cultured in the lab like normal bacteria because they require living hosts.

A nurse in an oncology clinic is collecting data from a client who has multiple myeloma. In relation to laboratory findings commonly associated with this diagnosis, the nurse should carefully observe the client for manifestations related to which of the following?

Thrombocytopenia

A nurse in an oncology clinic is conducting an assessment on a client with multiple myeloma. In relation to lab findings commonly associated with this diagnosis, the nurse should carefully assess the client for manifestations related to which of the following? a. hypocalcemia b. thrombocytopenia c. leukocytosis d. polycythemia

Thrombocytopenia

After receiving chemotherapy for lung cancer, a client's platelet count falls to 98,000/μl. What term should the nurse use to refer to this drop in the platelet count?

Thrombocytopenia

Patient is to begin taking clopidogrel(Plavix) to prevent stent stenosis what is the adverse reaction

Thrombocytopenia

A client is admitted to the hospital after being treated with vincristine for cancer. The client reports being diagnosed with peripheral neuropathy due to chemotherapy. When collecting data from the client, the nurse should expect to see which of the following clinical manifestations in the client's extremities?

Tingling

A nurse is caring for a client who has testicular cancer and is experiencing peripheral neuropathy as an adverse effect of chemotherapy. Which of the following client manifestations is an expected finding of peripheral neuropathy? Thinning of the scalp hair Tingling of the hands and feet Reduced ability to concentrate Sores in the mucous membranes

Tingling of the hands and feet

A client is admitting to the hospital after being treated with vincristine for cancer. The client report being diagnosed with peripheral neuropathy due to chemo. When assessing the client, the nurse should expect to see which symptom in the client's extremities? a. edema b. tingling c. ataxia d. spasms

Tingling.

A client diagnosed with leukemia asks the nurse why he must undergo bone marrow aspiration. Which response by the nurse is best?

To determine what type of leukemia you have; your physician will explain the procedure to you in more detail.

A client with metastatic cancer is experiencing neuropathic pain. Which alternative therapy is most beneficial in treating this type of pain?

Transcutaneous electrical nerve stimulation (TENS)

Absolute is defined as

True & Real

The spouse of a client with gastric cancer expresses concern that the couple's children may develop this type of cancer when they're older. When reviewing risk factors for gastric cancer with the client and family, the nurse explains that a certain blood type increases the risk by 10%. The nurse is referring to:

Type A.

USG measures what and is important for what?

Urine specific gravity - used to assess the ability of the renal tubules to concentrate or dilute the glomerular filtrate.

PVC direct from patient...

Use a red ring crit tube which contains Heparin

How do you differentiate between Rouleaux and Agglutination?

Use a saline wash...Rouleaux cells will wash away/apart easily (as if only held together by elmer's glue) but the Agglutination cells will not wash apart/away as if they are truly stuck together (super glue)

A nurse is providing preoperative teaching for a client who has colorectal cancer and is to undergo placement of a colostomy with a perineal wound. Which of the following statements by the client indicates and understanding of the teaching? "It will be a relief to not have any further rectal pain." "I will need to sit on a rubber donut when I am out of bed in the chair." "I can have only liquids for 2 days before the surgery." "The colostomy will start working about 7 days after the surgery."

"I can have only liquids for 2 days before the surgery." · The client should consume a full or clear liquid diet for 24 to 48 hr before the surgery to decrease bulk. The client should consume a low-residue diet for several days prior to surgery to decrease peristalsis.

A nurse is providing teaching to a client who has cancer and is receiving external radiation therapy. Which of the following statements by the client indicates an understanding of the teaching? "I need to protect the area from sunlight." "I'm going to apply a heating pad to the area after each treatment." "I'll massage the area once per day." "I'll wash the markings off after each therapy treatment."

"I need to protect the area from sunlight." · To prevent skin irritation and subsequent breakdown, the nurse should instruct the client to protect areas of skin from sunlight that receive radiation.

A nurse is providing discharge teaching to a client following open radical prostatectomy. The client is going home with an indwelling urinary catheter. Which of the following statements by the client indicates an understanding of the teaching? "I will be able to take a tub bath in 1 week." "I will change the catheter drainage bag once each week." "I will use suppositories to prevent constipation." "I will regain my bladder control once the catheter is removed."

"I will change the catheter drainage bag once each week."

A hospice nurse is providing education about palliative care to the partner of a client who has end-stage liver cancer. Which of the following statements by the partner indicates an understanding of teaching? "I will do my best to try to get him to eat something." "I will lay him flat if his breathing becomes shallow." "I will use an electric blanket to keep him warm." "I will continue to talk to him even when he's sleeping."

"I will continue to talk to him even when he's sleeping." · The nurse should reinforce to the partner that the client's hearing is thought to be the last sense to leave when in the dying process. Therefore, continue to softly communicate with the client.

A nurse on an oncology unit is providing discharge teaching to an adolescent female client who received a bone marrow transplant for leukemia. Which of the following information should the nurse include in the teaching? Select all that apply "Take your temperature twice each day." "You may return to school if you feel strong enough." "It is important to always wear shoes." "Clean your toothbrush weekly with isopropyl alcohol." "Avoid using tampons."

"Take your temperature twice each day." "It is important to always wear shoes." "Avoid using tampons." · Clients who are postoperative bone marrow transplants are immunosuppressed and should continually monitor for manifestations of infection. A temperature that is greater than 38° C (100° F) should be reported immediately to the provider. A client who had a bone marrow transplant is immunosuppressed and should wear shoes to prevent injury and decrease the risk for infection. The use of tampons is discouraged because they can disrupt the mucosal layer of the vagina and, if left in too long, can support the growth of bacteria.

A nurse is caring for a client who has breast cancer and is receiving a combination of chemotherapy medications. The client expresses confusion about the therapy. Which of the following explanations should the nurse provide? "The risk of renal toxicity is lessened when a combination of chemotherapy medications are used." "The chemotherapy medications act at different stages of cell division so more tumor cells are destroyed." "The use of more chemotherapy medications will shorten the time you have to be in treatment." "The combination of chemotherapy medications will eliminate the potential for bone marrow suppression."

"The chemotherapy medications act at different stages of cell division so more tumor cells are destroyed." · Different chemotherapeutic agents act at various stages of cellular mitosis (division). By combining agents, medication therapy is more effective in stopping or slowing the growth of cancerous cells by interfering with their ability to multiply.

A nurse is providing discharge teaching to a client who is postoperative following a right mastectomy for breast cancer. The client will be discharged with two Jackson-Pratt drains. Which of the following information should the nurse include in the teaching? "Empty the drainage tubes once per day." "Showering is permitted before the drainage tubes are removed." "The drainage tubes often are removed at the same time as the stitches." "Do not begin exercising the arm until the provider removes the drainage tubes."

"The drainage tubes often are removed at the same time as the stitches."

A nurse is providing postoperative discharge teaching to a client following a panhysterectomy for uterine cancer. Which of the following information should the nurse include in the teaching? "You will need to continue to use some form of birth control for 6 months." "You might experience manifestations of menopause." "Do not lift anything heavier than 15 pounds." "Pain or burning on urination is an expected outcome of this surgery."

"You might experience manifestations of menopause." · The nurse should inform the client that a panhysterectomy includes the removal of the uterus and the ovaries that might cause manifestations of menopause to occur. Manifestations of menopause include hot flashes, night sweats, and vaginal dryness.

Absolute observed retic count formula is:

# of retics/(divided by) 1000 X RBC ct. (10 6th RBC (this is an actual #) = retics per microliter - this is the only formula that we evaluate or compare. >60,000 retics/microliter is considered regenerative <60,000 retics/microliter is considered non-regenerative

Observed Retic Count formula is...

# retics /(divided by) 1000 X 100 = % retics rounded to tenth place

(SELECT ALL THAT APPLY) A client receiving chemotherapy for breast cancer develops myelosuppression. Which instructions should the nurse include in the discharge teaching plan?

(1) Avoid people who have recently received attenuated vaccines. (2) Avoid activities that may cause bleeding. (3) Wash hands frequently. (5) Avoid crowded places, such as shopping malls.

(SELECT ALL THAT APPLY) A client in the terminal stage of cancer is being transferred to hospice care. Which information should the nurse include in the teaching plan regarding hospice care?

(1) Care focuses on controlling symptoms and relieving pain. (2) A multidisciplinary team provides care. (5) Bereavement care is provided to the family.

(SELECT ALL THAT APPLY) After having a lobectomy for lung cancer, a client receives a chest tube connected to a three-chamber chest drainage system. The nurse observes that the drainage system is functioning correctly when she notes which of the following?

(1) Fluctuations in the water-seal chamber occur when the client breathes. (3) Intermittent bubbling occurs in the water-seal chamber. (4) Gentle bubbling occurs in the suction control chamber. (5) Drainage is collecting in the drainage chamber

(SELECT ALL THAT APPLY) What diagnostic studies are recommended to be performed annually on all women older than age 40?

(1) Mammography (4) Clinical breast examination

(SELECT ALL THAT APPLY) After undergoing chemotherapy to treat breast cancer, a client has a white blood cell (WBC) count of 1,300/microliter. Which nursing interventions should be used with this client?

(1) Monitor the client's temperature and vital signs; report temperature higher than 101.3° F (38.5° C). (2) Make sure no one with a viral or bacterial infection enters the client's room. (3) Administer hematopoietic growth factors as prescribed. (4) Reinforce good hand-washing techniques to anyone entering the client's room.

(SELECT ALL THAT APPLY) A client with laryngeal cancer has undergone laryngectomy and is receiving radiation therapy to the head and neck. The nurse should monitor the client for which adverse effects of external radiation?

(1) Xerostomia (2) Stomatitis (5) Dysgeusia

(SELECT ALL THAT APPLY) A client with bladder cancer undergoes surgical removal of the bladder and construction of an ileal conduit. Which data collection findings indicate that the client is developing complications?

(2) The stoma appears dusky. (3) The stoma protrudes from the skin. (6) The client experiences sharp abdominal pain and rigidity.

A health care professional is caring for a patient who is about to begin iron dextran (INFeD) therapy. When administering the drug IM, the health care professional should take which of the following actions? (select all) - Administer a test dose - Monitor BP - Have epinephrine available - premedicate with atropine - use the Z-track technique

- Administer a test dose - Monitor BP - Have epinephrine available - use the Z-track technique

A patient is about to begin therapy with a recombinant factor IX (BeneFix) to treat hemophilia B. The patient asks the health care professional about the risk of disease transmission with recombinant factor IX, as compared with plasma-derivded factor IX. The health care professional should explain that recombinant factor IX eliminates the risk for which of the following? - HIV - Cytomegalvirus - Creutzfeldt-Jacob disease - Anaphylaxis

- Creutzfeldt-Jacob disease

A patient who is taking ferrous sulfate (Feosol) to treat iron-deficiency anemia develops severe nausea and vomiting due to iron toxicity. Which of the following drugs should the health care professional use to treat this complication? - flumazenil (Mazicon) - acetylcysteine (Acetadote) - Naloxone - Deferoxamine (Desferal)

- Deferoxamine (Desferal)

A health care professional administers epoetin alfa (Epogen) subcutaneously to a patient who has renal failure. The health care professional should monitor the patient for which of the following adverse effects? - HTN - muscle pain - edema - dry mouth

- HTN

A health care professional is caring for a patient who is about to begin taking epoetin alfa (Epogen). Which of the following laboratory values should increase with effective therapy? - PT - WBC - Hgb - Platelets

- Hgb

A health care professional is caring for a patient who is about to begin taking warfarin (Coumadin). The health care professional should caution the patient about taking which of the following over-the-counter drugs? - diphenhydramine - dimenhydrinate (Dramamine) - Calcium carbonate (Tums) - Ibuprofen (Motrin)

- Ibuprofen (Motrin)

A patient who is taking warfarin (Coumadin) arrives at the emergency department and reports rectal bleeding. Which of the following drugs should the health care professional have available? - filgrastim (Neupogen) - deferoxamine (Desferal) - Protamine - Vitamin K

- Vitamin K

A health care professional is caring for a patient who is about to begin alteplase (Activase) therapy to treat pulmonary embolism. Which of the following drugs should the health care professional have available in the event of a severe adverse reaction? - vitamin K - aminocaproic acid (Amicar) - Protamine - Deferoxamine (Desferal)

- aminocaproic acid (Amicar)

A health care professional is preparing to administer the prescribed dose of filgrastin (Neupogen) to a patient. The patient's laboratory results indicate that the patient has a WBC count of 12.000/mm^3. Which of the following actions should the health care professional take? - clarify the prescription with the provider - administer the drug with an analgesic - limit venipunctures - assess breath sounds

- clarify the prescription with the provider (stop therapy w/ WBC above 10,000)

A health care professional is caring for a patient who is about to begin taking oprelvekin (Neumega) to treat thrombocytopenia. The health care professional should tell the patient to report which of the following indications of an adverse reaction? (select all) - conjunctivitis - tinnitus - swollen ankles - jaundice - palpitations

- conjunctivitis - swollen ankles - palpitations

A health care professional is caring for a patient who is about to begin taking ferrous sulfate (Feosol) to treat iron-deficiency anemia. When talking with the patient about the drug, the health care professional should include which of the following instructions? (select all) - eat iron-enriched foods - spread the dosage across each day - take the drug on an empty stomach -report dark or black stools - increase dietary fiber intake

- eat iron-enriched foods - spread the dosage across each day - take the drug on an empty stomach - increase dietary fiber intake

A health care professional is caring for a patient who has mild hemophilia A and is about to begin taking desmopressin (DDAVP) to prevent bleeding. The health care professional should monitor for which of the following adverse reactions? - weight loss - edema - polyuria - tachycardia

- edema

A health care professional is caring for a patient who is about to begin taking filgrastin )Neupogen) to treat neutropenia. The health care professional should assess the patient for which of the follwoing adverse effects? - dusky nail beds - petechiae - enlarged spleen - swollen calf

- enlarged spleen (w/ long-term use, monitor for abdominal pain)

A health care professional is caring for a patient who has a known sensitivity to E. coli-derived proteins. This sensitivity is a contraindication for which of the following drugs? - epoetin alfa (Erythropoietin) - oprelvekin (Neumega) - filgrastim (Neupogen) - alteplase (Activase)

- filgrastim (Neupogen)

A health care professional is caring for a patient who is undergoing bone marrow transplantation following high-dose chemotherapy. Which of the following drugs should the health care professional expect the primary care provider to prescribe to reduce the patient's risk of infection? - epoetin alfa (Erythropoietin) - oprelvekin (Neumega) - filgrastim (Neupogen) - alterplase (Activase)

- filgrastim (Neupogen)

A health care professional is caring for a patient who is about to begin factor VIII (Advate) therapy to treat hemophilia A. When administering factor VIII, which of the following actions should the health care professional take? - have emergency equipment ready - premedicate with aspirin (Ecotrin) - administer via rapid IV bolus - administer the powdered form orally

- have emergency equipment ready

A health care professional is caring for a patient who is about to begin alteplase (Activase) therapy. The health care professional should monitor the patient for which of the following indications of a possible adverse effect? - hives - headache - edema - HTN

- headache

A health care professional is caring for a patient who is about to begin taking oprelvekin (Neumega). The health care professional should explain to the patient that the drug has which of the following effects? - stimulates leukocyte maturation - causes myelosuppression - increases platelet production - destroys blood clots

- increases platelet production

A health care professional is caring for a patient who is about to begin taking aspirin (Ecotrin) to reduce the risk of a cardiovascular event. The health care professional should understand that the drug inhibits platelet aggregation by which of the following mechanisms? - activating thromboxane A2 - blocking adenosine diphosphate receptor agonists - suppressing specific clotting factors - inhibiting cyclooxygenase action in platelets.

- inhibiting cyclooxygenase action in platelets.

A health care professional is administering epoetin alfa (Epogen) subcutaneously to a patient who has renal failure. The health care professional should take which of the following actions? - shake the vial - insert the needle into the vial once - dilute the drug first with normal saline - save the used vial for the next dose

- insert the needle into the vial once

A health care professional is caring for a patient who is about to begin taking clopidogrel (Plavix) to prevent thrombus formation. The health care professional should question the use of clopidogrel by a patient who has which of the following? - myocardial infarction - peptic ulcer disease - pancreatitis - myasthenia gravis

- peptic ulcer disease

A health care professional is caring for a patient who is about to begin taking folic acid to treat megaloblastic anemia. The health care professional should monitor which of the following laboratory values? - amylase level - reticulocyte count - C-Reactive protein - creatinine clearance

- reticulocyte count

A health care professional is caring for a patient who is about to begin taking clopidogrel (Plavix) to prevent stent stenosis. The health care professional should monitor the patient for which of the following adverse reactions? - thrombocytopenia - hyponatremia - lymphocytopenia - uricemia

- thrombcytopenia

A health care professional should assess a patient who has megaloblastic anemia for indications of which of the following vitamin deficiencies? - vitamin B12 - vitamin K - vitmain C - vitamin D

- vitamin B12

A health care professional is caring for a patient who is about to begin taking alteplase (Activase) to treat acute myocardial infarction. The health care professional should understand the drug is most effective when the patient receives it - 24 hr after the clot formation - after initiation of anticoagulation - prior to clot formation - within 3 hr of symptom onset

- within 3 hr of symptom onset (As soon as possible)

Give a specific example of each of these 3 components and a function of each:

...

Patient is to begin alteplase therapy to treat pulmonary embolism and what should be available for the severe adverse reaction

...aminocaproic acid (amicar)

Patient is to begin taking our Oprelvekin (neumega) to treat thrombocytopenia what should the patient report as an indication of adverse reaction

...conjunctivitis swollen ankles palpitations

A healthcare professional is caring for a patient who is about to begin taking filgrastim to treat neutropenia the healthcare professional should assess the patient for which of the following adverse affects

...enlarged spleen

Patient is to begin taking oprelvekin what should the healthcare professional explain about The following affects

...increases platelet production

Normal Creatinine values for a dog are: Normal Creatinine values for a cat are:

0.5-1.8 mg/dl 0.8-2.4 mg/dl

Whole blood is composed of 3 basic components:

1. 2. 3.

There are 2 proteins which dominate & make up the bulk of the total protein measurement and they are?

1. Albumin - responsible for osmotic pressure (Made in Liver). Think "blood dumpling" holding H2O in (preventing its diffusion into the body) & maintaining fluid volume in vasculature 2. Immunoglobulins - inflammation, immunity & defense. (Liver & Lymph Tissue)

IMHA (Immune Mediated Hemolytic Anemia) is a huge & generic category) What are some "more real" causes of IMHA?

1. Auto-Immune Hemolytic Anemia (Idiopathic) 2. Mycoplasma hemofaelis (caused by intracellular RBC organisms) bacteria are parasitic, contain flagella 3. Heinz Bodies - are inclusions within red blood cells composed of denatured hemoglobin. There is no specific treatment for Heinz bodies; however they are important as a diagnostic indicator, Heinz bodies can also be found in chronic liver disease.

The three parts of a blood film are...

1. Body 2. Monolayer 3. Feathered Edge

What disadvantages are there in using heparinized blood?

1. Causes "hazy" staining with routine differential stains 2. Expensive 3. WBC's may clump

What are 3 trends of RBC maturation?

1. Cells become smaller 2. Loss of nucleus 3. Basophilic ~ Eosinophilic

List the 3 trends of RBC maturation:

1. Cells go from basophilic to eosinophilic 2. Cells go from larger to smaller 3. Cells go from nucleated to a-nucleated

List all the steps for making a reticulocyte count: 1. Sample used: 2. Incubation time: 3. Make & Examine: 4. Magnification: 5. Counting procedure:

1. Equal parts EDTA whole blood & NMB 2. 10-20 minutes (20 preferred) 3. Blood film 4. 100x 5. Count 1000 RBC's, tally retics

Explain how to stain a sample with Diff Quik: What are the contents?

1. Fixative - Dip 5 times for 1 second each 2. Eosinophilic - Dip 5 times for 1 second each 3. Basophilic - Dip 7-10 times for 1 second each

What are the three common abnormalities of plasma?

1. Hemolytic [pink/red] (contain broken red blood cells/RBC's) 2. Icteric [yellow] (indicates hyperbilirubenemia) 3. Lipemic [white] (indicated fat) Must be quantified as MILD, MODERATE or MARKED

What is Secondary Absolute Polycythemia? Secondary is abbreviated as 2*

1. Hypoxia w/out anemia - Lungs inability to absorb O2 but RBC measurements are totally normal. Usually from altitude change, Asthma, COPD (Emphysema) Something wrong with the lungs 2. Glucocorticoids - No EPO involved, increased steroid use over time causes RBC's to increase (this is not an appropriate body response)

Name the cancer associated with the following virus/bacteria: 1. Hep B & C 2. HIV 3. HPV 4. H. Pylori 5. Epstein-Barr 6.T-cell leukemia virus

1. Liver cancer 2. lymphoma and Kaposi's sarcoma 3. cervical cancer 4. stomach cancer and lymphoma 5. lymphoma 6. leukemia and lymphoma

Evaluation of RBC status has 4 steps:

1. MCV formula - does result indicate patient is Macrocytic (large), Microcytic(small) or Normocytic (normally sized)? 2. MCHC formula - does result indicate patient is Hyperchromic (Too much hemoglobin), Hypochromic (too little hemoglobin, or Normochromic (just right amount)? 3. Absolute retic count - does patient result indicate Regenerative or Non-regenerative? 4. RBC, PCV, & Hb testing together - do results indicate a decrease in RBC's or Hb? If so this is the definition of ANEMIA

What advantages are there in using Heparin as an anticoagulant?

1. Maintains size and shape of RBC's even if poor blood to anticoagulant ratio 2. Best for blood gas reading & hormone & drug levels in blood 3. Used for reptile hematologic studies

List some of the errors in the collection of blood for a CBC (in LTT) causing clots (completely invalidating the CBC)

1. Not mixing sample gently and thoroughly 2. Not enough EDTA and too much blood 3. "Dinking around" (not preparing in a timely fashion) (slow blood draw) 4. Traumatic venipuncture (initiates clotting cascade) 5. High fibrinogen levels

Conditions in the body and the resulting changes to PCV and TP:

1. Overhydration lowers relative PCV and TP (total protein) 2. Dehydration raises relative PCV and TP (total protein) 3. Liver failure maintains PCV @ normal and lowers the absolute total protein value 4. Hemorrhage lowers the absolute PCV and lowers the absolute total protein 5. Renal failure (kidney makes EPO) lowers the absolute PCV & may raise, lower or stay normal for total protein 6. Hemolysis will lower the absolute PCV & increase the artifactual Total Protein (TP) 7. Bone Marrow Toxin will lower the absolute PCV but the total protein (TP) will remain within normal limits

RBC's are removed from the blood stream (body) by 2 mechanisms...name them.

1. PRIMARY WAY IS VIA Macrophages in the liver, (Mononuclear Phagocytic System-MPS) spleen and bone marrow which consume old RBC's and recycle the parts. RBC's are broken down into HEMO & GLOBULIN. The iron is stored, the heme pigment converted into bilirubin (out via the liver) and excreted. The globin is broken down into protein This is Extravascular Hemolysis which occurs normally. 2. Secondary way is by Intravascular Hemolysis (not very common) Done by monocytes in the peripheral blood which can cause a hemolytic sample.

What are the two types of reticulocytes? In what species are retics never counted?

1. Punctates 2. Aggregates The horse (polychromatophilic cells are never released early in this species so there won't be any retics to count)

Name the tests which comprise the CBC?

1. Red Blood Cell Count (RBC's available to the body) 2. White Blood Cell Count (WBC's overall amount) 3. Platelet Count - amount of platelets present 4. Packed Cell Volume - (PCV) % quantity of blood made up of RBC's 5. Total Protein: (TP) Amount of protein in fluid portion of blood 6. Hemoglobin: Amount of hemoglobin present in RBC's 7. Differential Smear (Diff) - Done on Blood Film

List some "special " considerations of working with EDTA blood:

1. Refrigerate blood if not able to run tests immediately as this will minimize cell damage 2. Rotate tube gently & thoroughly to prevent clotting and hemolysis 3. Make blood film ASAP

What are the 6 stages of RBC development:

1. Rubriblast - large cell, large nucleus, open nuclear cytoplasm & open nuclear chromatin (starts very dark purple nucleus) 2. Prorubricyte - slightly smaller, slightly less purple nucleus, open chromatin, no nucleoli 3. Rubricyte - smaller, chromatin getting tighter 4. Metarubricyte - cell is smaller still, chromatin dark & dense 5. Polychromatophilic - no nucleus, small & purple 6. Mature RBC - "bag of hemoglobin" - cell has become completely red

List some of the errors in the collection of blood for a CBC (in LTT) causing Hemolysis (broken cells)

1. Shaking 2. To small of a needle, too much back pressure 3. Squirting blood into the tube 4. Spinning sample too long or too fast 5. Water in the syringe 6. Freezing & Thawing 7. In rocker tray too long

What are some disadvantages of using Sodium Citrate as an anticoagulant?

1. Shrinks cells 2. Tube must be full, ratio of blood and anticoagulant is crucial

What are some advantages of using Sodium Citrate as an anticoagulant?

1. Used for most coagulation studies needing anticoagulated blood 2. Non-toxic & can be used for blood transfusions

Explain how to stain a sample with Wright's Stain: What are the contents?

1. Wright's Stain - 1-3 minutes 2. Buffer w/ 1 pipette Wright's Stain - 1 minute 3. Distilled H2O - 7-10 1 second dips

Name the cancer type for each cell type affected: 1. epithelial 2. glandular organs 3. mesenchymal tissue 4. blood-forming cells 5. lymph 6. plasma fcells

1. carcinoma 2. adenocarcinoma 3. sarcoma 4. leukemia 5. lymphoma 6. myeloma

USG normal range for a dog is: USG normal range for a cat is:

1.025 1.045

A "normal" hemoglobin measurement would be

1/3 of PCV (packed cell volume) reported out to tenths place in g/dl

A client is hospitalized with lung cancer. To manage severe pain, the physician prescribes a continuous I.V. infusion of morphine. Which formula should the nurse use to check that the morphine dose is appropriate for the client?

10 mg/70 kg of body weight

Normal Na (Sodium) ranges for a dog are: Normal Na (Sodium) ranges for a cat are:

144-160 mEq/l (milequivalents per liter) 150-165 mEq/l (milequivalents per liter)

Alk Phosphate normal range for dog is: Alk Phosphate normal range for cat is:

2 - 95 IU/L 0-62 IU/L

A client with a history of colon cancer has a permanent colostomy. The nurse must irrigate the colostomy to prepare the client for diagnostic testing. When irrigating, how far into the stoma should the nurse insert the lubricated catheter?

2" to 4"

Platelet normal values for a dog are: Platelet normal values for a cat are:

200,000-500,000 per microliter ROUND TO THE WHOLE CELL 200,000-500,000 per microliter

Potassium (K) levels in dogs are: Potassium (K) levels in cats are:

3.5-5.8 mEq/l (milequivalents per liter) 3.5-5.8 mEq/l (milequivalents per liter)

Normal MCHC values for a dog are: Normal MCHC values for a cat are:

30-36 g/dl (grams per deciliter) ROUND TO THE NEAREST 1/10TH 30-36 g/dl (grams per deciliter)

PCV (Packed Cell Volume) normal lab values for a dog are: PCV (Packed Cell Volume) normal lab values for a cat are:

37-55% ROUND TO THE WHOLE % 30-45%

Which client has the highest risk of ovarian cancer?

45-year-old woman who has never been pregnant

RBC (Red Blood Cell) normal lab values for dog are: RBC (Red Blood Cell) normal lab values for cat are:

5-10 10/6 ul (microliters) ROUND TO THE 1/100th 5-11 10/6 ul (microliters)

How long does it take to see a response by the body to anemia in the peripheral blood?

5-7 days

Total protein values for a dog are: Total protein values for a cat are:

5.0-7.0 g/dl MEASURED TO THE 2/10TH (EVEN NUMBER) Measure on a refractometer 5.0-8.0 g/dl

Normal RBC Morphology for a horse:

5.7um, Rouleaux, lack central pallor, report/quantify Rouleaux

Normal RBC Morphology for a cat:

5.8um, mild anisocytosis, inconsistent central pallor, occasional Heinz Bodies. No need to mention central pallor, report/quantify anisocytosis, crenation & Heinz Bodies

WBC (White Blood Cell) normal lab values for dog are: WBC (White Blood Cell) normal lab values for cat are:

6 -17 K/ul (microliter) ROUND TO THE WHOLE CELL 5.5-19.5 K/ul (microliter)

Normal MCV (Mean Cell Volume) values for a dog are: Normal MCV (Mean Cell Volume) values for a cat are:

60-70 fl (fentoliters) ROUND TO NEAREST 1/10TH 39-55 fl (fentoliters)

BUN values for a dog are: BUN values for a cat are:

7-27 mg/dl 16-36 mg/dl

Glucose normal values for a dog are: Glucose normal values for a cat are:

77-125 mg/dl 76-145 mg/dl

Normal RBC Morphology for a dog:

7um, biconcave discs, consistent central pallor. If your dog is NOT anemic & you are not seeing central pallor report out on lab form - Mild,Moderate or Marked inconsistent central pallor (ICP)

ALT normal values for a dog are: ALT normal values for a cat are:

9-80 IU/L 7-67 IU/L

Needle size from smallest to largest: The length of the needle depends upon the species...

<25 gauge (black) is smallest to 16 gauge (gray) which is largest typically used Birds/Small Pets: 3/8 - 1/2" long Most Dogs/Cats: 3/4-1" long Large Animals: 1-3" long 16 ga needle (gray) for anterior vena cava stick in swine is 3-3 1/2 " long

Retics observed normal values for a dog are: Retics observed normal values for a cat are:

>1% ROUND TO NEAREST 1/10TH >1%

Absolute retic normal values for a dog are: Absolute retic normal values for a cat are:

>60,000 per microliter - evaluated as REGENERATIVE ROUND TO THE WHOLE CELL >60,000 per microliter - evaluated as REGENERATIVE If the above value is <60,000 the evaluation is considered NON-REGENERATIVE...this means the patient is NOT able to make its own red blood cells and this can be due to numerous causes.

A nurse is assessing a client who has chronic neutropenia and who has bee receiving filgrastim. Which of the following actions should the nurse take to assess for an adverse effect of filgrastim? A. Assess for bone pain. B. Assess for right lower quadrant pain. C. Auscultate for crackles in the bases of the lungs. D. Auscultate the chest to listen for a heat murmur.

A

A nurse is preparing to administer a transfusion of 300 mL of pooled platelets for a client who has severe thrombocytopenia. The nurse should plan to administer the transfusion over which of the following time frames? A. Within 30 min/unit B. Within 60 min/unit C. Within 2 hr/unit D. Within 4 hr/unit

A

A nurse is assessing a client who is postoperative and has anemia due to excess blood loss following surgery. Which of the following findings should the nurse expect? a. Fatigue b. Hypertension c. Bradycardia d. Diarrhea

A The nurse should identify that the client who has anemia due to blood loss following surgery will experience fatigue. This is due to the body's decreased ability carry oxygen to vital tissues and organs. -- Constipation is a manifestation of anemia due to blood loss following surgery. Tachycardia is manifestation of anemia due to blood loss following surgery. Hypotension is manifestation of anemia due to blood loss following surgery.

WBC's are...

A WBC count is a blood test to measure the number of white blood cells (WBCs). White blood cells help fight infections. They are also called leukocytes. There are five major types of white blood cells: Basophils Eosinophils Lymphocytes (T cells and B cells) Monocytes Neutrophils

A client with suspected lung cancer is scheduled for thoracentesis as part of the diagnostic workup. The nurse reviews the client's history for conditions that might contraindicate this procedure. Which condition is a contraindication for thoracentesis?

A bleeding disorder

BUN is...

A blood urea nitrogen (BUN) test reveals important information about how well your kidneys and liver are working. A blood urea nitrogen test measures the amount of urea nitrogen that's in your blood. Your liver produces ammonia — which contains nitrogen — after it breaks down proteins used by your body's cells. The nitrogen combines with other elements, such as carbon, hydrogen and oxygen, to form urea, which is a chemical waste product. The urea travels from your liver to your kidneys through your bloodstream. Healthy kidneys filter urea and other waste products from your blood. The filtered waste products leave your body in urine.

A registered nurse (RN) on the oncology floor is busy with one client, so she delegates care of another client to her coworker, a licensed practical nurse (LPN). The client that the LPN begins caring for requires a 3-hour chemotherapy infusion. Which of the following actions is in accordance with her state's nurse practice act?

A chemotherapy-certified RN must begin the chemotherapy infusion; then the LPN may monitor the client.

During a breast examination, which finding most strongly suggests that the client has breast cancer?

A fixed nodular mass with dimpling of the overlying skin

A client with multiple myeloma is admitted to the unit with a whiteblood cell count of 2,200/mm3. Which of the following foods should the nurse prohibit family members from bringing to the client?

A fresh fruit basket

A nurse is admitting a client who has multiple myeloma and a white blood cell count of 2,200/mm3. Which of the following foods should the nurse prohibit the family members from bringing to the client? Fried chicken from a fast food restaurant A case of canned nutritional supplements A factory-sealed box of chocolates A fresh fruit basket

A fresh fruit basket · Raw fruits and vegetables are contraindicated for a client who has neutropenia, as the skin might harbor bacteria that can cause an infection. The nurse should prohibit these foods from entering the client's room.

"A few minutes after beginning a blood transfusion, the nurse notes that the client has chills, dyspnea, and urticaria. The nurse reports this to the physician immediately because the client probably is experiencing:

A hemolytic allergic reaction caused by an antigen reaction.

Leukocytosis

A high number of WBCs ...It may be due to: Anemia Bone marrow tumors Infectious diseases Inflammatory disease (such as rheumatoid arthritis or allergy) Leukemia Severe emotional or physical stress Tissue damage (for example, burns)

Leukopenia is...

A low number of WBCs...may be due to: Bone marrow deficiency or failure (for example, due to infection, tumor, or abnormal scarring) Collagen-vascular diseases (such as systemic lupus erythematosus) Disease of the liver or spleen Radiation therapy or exposure

FALSE

A newborn will have a relative hypoprotenemia

T or F?

A traumatic venipuncture will cause a hemolytic sample?

A nurse is transfusing a unit of packed red blood cells (PRBCs) for a client who has anemia due to chemotherapy. The client reports a sudden headache and chills. The client's temperature is 2 degrees F higher than her baseline. In addition to notifying the provider, which of the following actions should the nurse take? (Select all that apply) A. Stop the transfusion B. Place the client in an upright position with feet down C. Remove the blood bag and tubing from the IV catheter D. Obtain a ruined specimen E. Infuse dextrose 5% in water through the IV.

A, C, D

Total Protein Measurement is.... Why do we evaluate the Total Protein?

A-Cellular (non-cellular) dissolved substances in the fluid portion of blood. (EX: enzymes) Liver function, TP will be low if liver is not functioning correctly, also checks on hydration status

A nurse us preparing to administer 2units of packed RBCs to an older adult client. Which of the ff. actions should the nurse take? a. Administer each unit over 3 hr. b. Use an 18-gauge needle to obtain venous access. c. Use blood that is less than a month old. d. Obtain the client's vital signs every 30min throughout the transfusion.

A. The nurse should administer blood to an older adult client at a slower rate. Therefore, each unit should be administered over 2 to 4 hr. The nurse should use no larger than a 19-gauge needle to obtain venous access on an older adult client. The nurse should use blood that is less than a week old for older adult clients. Older blood cell membranes are more fragile and can break, releasing potassium into circulation. The nurse should obtain an older adult client's vital signs every 15 min throughout the transfusion.

A nurse is monitoring a client who is on telemetry. Which of the ff. findings on the ECG strip should the nurse recognize as normal sinus rhythm? a. The P wave falls before the QRS complex. b. The T wave is in the inverted position. c. The P-R interval measures 0.22 seconds. d. The QRS duration is 0.20 seconds.

A. The nurse should recognize that in normal sinus rhythm the P wave, representing atrial depolarization, falls before the QRS wave. In normal sinus rhythm, the T wave is upright. In normal sinus rhythm, the P-R interval has a constant duration between 0.12 and 0.20 seconds. In normal sinus rhythm, a QRS has a constant duration between 0.04 and 0.10 seconds.

A nurse us preparing to administer 2units of packed RBCs to an older adult client. Which of the ff. actions should the nurse take? a. Administer each unit over 3 hr. b. Use an 18-gauge needle to obtain venous access. c. Use blood that is less than a month old. d. Obtain the client's vital signs every 30min throughout the transfusion.

A. The nurse should administer blood to an older adult client at a slower rate. Therefore, each unit should be administered over 2 to 4 hr. -- The nurse should use no larger than a 19-gauge needle to obtain venous access on an older adult client. The nurse should use blood that is less than a week old for older adult clients. Older blood cell membranes are more fragile and can break, releasing potassium into circulation. The nurse should obtain an older adult client's vital signs every 15 min throughout the transfusion.

A nurse is monitoring a client who is receiving a blood transfusion. Which of the ff finding indicates an allergic transfusion reactions? a. generalized urticaria b. Blood pressure 184/92 mm Hg c. Distended jugular veins d. Bilateral flank pain.

A. The nurse should recognize urticaria as an indicator of an allergic transfusion reaction. Other clinical manifestations include itching and signs of anaphylaxis with bronchospasm. Hypertension may be an indication of circulatory overload rather than an allergic reaction. Distended jugular veins may be an indication of circulatory overload rather than an allergic reaction. Bilateral flank pain may be an indication of a hemolytic transfusion reaction rather than an allergic reaction.

A nurse is monitoring a client who is receiving a blood transfusion. Which of the ff finding indicates an allergic transfusion reactions? a. generalized urticaria b. Blood pressure 184/92 mm Hg c. Distended jugular veins d. Bilateral flank pain.

A. The nurse should recognize urticaria as an indicator of an allergic transfusion reaction. Other clinical manifestations include itching and signs of anaphylaxis with bronchospasm. -- Hypertension may be an indication of circulatory overload rather than an allergic reaction. Distended jugular veins may be an indication of circulatory overload rather than an allergic reaction. Bilateral flank pain may be an indication of a hemolytic transfusion reaction rather than an allergic reaction.

FALSE

Absolute hyperproteinemia can be caused by liver failure

RBC Descriptions: Protrusions occur on RBC, long & irregular in length

Acanthocyte

In assessing a postmastectomy client, the nurse determines that the client is in denial. The nurse can best respond by:

Accepting the denial.

A nurse is working with a dying client and his family. Which communication technique is most important to use?

Active listening

"A client is prescribed tamoxifen (Nolvadex), 20 mg by mouth twice per day for treatment of breast cancer. The client complains to the nurse that she has worsening bone pain. How should the nurse respond?

Acute worsening of bone pain commonly indicates that the drug will produce a good response.

A healthcare professional is caring for a patient who is about to begin iron dextran therapy when administering the drug a healthcare professional should take which of the following actions

Administer a test dose Monitor blood pressure have epinephrine available uses the z track technique

A nurse in an emergency department is preparing to administer alteplase accelerated therapy to a client who is having a myocardial infarction. Which of the following actions should the nurse plan to take? (Select all that apply) a. Administer the medication within 30min of the client's arrival to the department b. Reconstitute the medication with sterile Water. c. Administer a 15 mg IV bolus d. Tell the client that the purpose of the medication is to keep a new clot from forming. e. Assess the client for back pain.

Administer the medication within 30 min of the client's arrival to the department. The benefits of alteplase are greatly increased when administered as early as possible. Current recommendations are that the client is given alteplase within 30 min of arrival in the emergency department. Research shows a decrease in mortality and a reduction in the size of the infarction when alteplase is administered within 30 min. Reconstitute the medication with sterile water is correct. Alteplase is available as a powder. The nurse should use sterile water to reconstitute the medication. Administer the medication in a 15 mg IV bolus is correct. The nurse should administer an accelerated, or loading dose, to promote a rapid therapeutic effect of the medication. Tell the client that the purpose of the medication is to keep a new clot from forming is incorrect. The nurse should tell the client that the purpose of the medication is to dissolve the existing clot. The nurse will also administer anticoagulants to reduce the risk of new clot formation. Assess the client for back pain is correct. The nurse should assess the client for indications of bleeding, which include report of back pain, headache, changes in level of consciousness, and decreased levels of hematocrit and hemoglobin.

What are some disadvantages of EDTA anticoagulant?

After 6 hrs. WBC's begin to break down and disintegrate After 4 hrs. platelets swell and clump Incorrect blood to EDTA ratio causes cells to shrink

Which of the following is a risk factor associated with prostate cancer?

Age older than 40

A client with breast cancer is receiving a combination of chemotherapy agents. The nurse understands that the rationale for this combined treatment modality is that the

Agents act a different stages of cellular mitosis

A client with breast cancer is receiving a combo of chemo agents. The nurse IDs the rationale for this combined treatment modality is that the- a. risk of renal toxicity is lessened b. agents act at different stages of cellular mitosis c. length of treatment will be shorter d. potential for bone marrow suppression is eliminated

Agents act at different stages of cellular mitosis.

ALT is what?

Alanine transaminase (ALT) is an enzyme found in the highest amounts in the liver. Injury to the liver results in release of the substance into the blood. Increased levels of ALT often means that liver disease is present. Liver disease is even more likely when levels of other liver blood tests are also increased.

"A client has an abnormal result on a Papanicolaou test. After admitting that she read her chart while the nurse was out of the room, the client asks what dysplasia means. Which definition should the nurse provide?

Alteration in the size, shape, and organization of differentiated cells

Relative is defined as...

Amount is the same, it just looks/presents differently (most of the time a fluid type change) because the concentration has changed. EX: FISH IN THE POND

RBC's are...

An RBC count is a blood test that tells how many red blood cells (RBCs) you have. RBCs contain hemoglobin, which carries oxygen. How much oxygen your body tissues get depends on how many RBCs you have and how well they work.

Hyperproteinemia is

An abnormally high concentration of protein in the blood

What does it mean if a cell is Hypochromic?

An anemic condition in which the percentage of hemoglobin in the red blood cells is abnormally low

FALSE

An animal with a new anemia would have a macrocytic hypochromic non-regenerative anemia

FALSE

An animal with kidney failure would have a reticulocytosis

FALSE

An animal with liver failure will have an absolute hypoproteinemia and an absolute decrease in RBC's

Polycythemia is defined as...

An increase in the # of RBC's (Red Blood Cells)

Lower than normal RBC counts can be caused by:

Anemia Bone marrow failure (for example, from radiation, toxins, or tumor) Erythropoietin deficiency (secondary to kidney disease) Hemolysis (RBC destruction) due to transfusion, blood vessel injury, or other cause Hemorrhage (bleeding) Leukemia Malnutrition Multiple myeloma Nutritional deficiencies of: Iron Copper Folate Vitamin B-12 Vitamin B-6 Overhydration Pregnancy

anemia & hypoxia:

Anemia - think RBC production Hypoxia - think O2 production Often the two go hand in hand but you can be hypoxic and have plenty of RBC's

RBC Descriptions: RBC's of Unequal Size

Anisocytosis

Hogs venipuncture sites & needle sizes

Anterior Vena Cava (AVC) - direct stick with a 16 ga (gray) with 3 - 3 1/2 inch long needle

A client complains of sporadic epigastric pain, yellow skin, nausea, vomiting, weight loss, and fatigue. Suspecting gallbladder disease, the physician orders a diagnostic workup, which reveals gallbladder cancer. Which nursing diagnosis may be appropriate for this client?

Anticipatory grieving

To combat the most common adverse effects of chemotherapy, the nurse would administer an:

Antiemetic.

A nurse anticipates that a client who is receiving antineoplastic medications may manifest what side effects? a. gingival hyperplasia b. hirsutism c. aplastic anemia d. weight gain

Aplastic Anemia: it is a sign of bone marrow suppression, which is a common side effect of chemotherapeutic agents. Aplastic anemia results in pancytopenia, a decrease in white blood cells, red blood cells, and platelet count.

A nurse anticipates that a client who is receiving antineoplastic medications may manifest which of the following side effects?

Aplastic anemia

The nurse is caring for a client receiving the fentanyl transdermal system (Duragesic-50) for pain management. When applying a new system, the nurse should:

Apply the system immediately after removal from a package.

TRUE

As a newborn ingests colostrum, TP goes up but not into normal range

The ABCD method offers one way to assess skin lesions for possible skin cancer. What does the A stand for?

Asymmetry

What are the "average" PCV numbers for a dog and a cat for use in the corrected observed retic count formula?

Average PCV for dog = 45% Average PCV for cat = 35%

A client receiving external radiation to the left thorax to treat lung cancer has a nursing diagnosis of Risk for impaired skin integrity. Which intervention should be part of this client's plan of care?

Avoiding using soap on the irradiated areas

A nurse is planning to administer IV alteplase to a client who is demonstrating manifestations of a massive pulmonary embolism. Which of the following interventions should the nurse plan to take? A. Administer IM enoxaparin along with the alteplase dose. B. Hold direct pressure on the puncture sites for up to 30 minutes. C. Administer aminocaproic acid IV prior to alteplase infusion. D. Prepare to administer alteplase within 8 hour of manifestation onset.

B

A nurse is preparing to transfuse a unit of packed red blood cells for a client who has severe anemia. Which of the following interventions will prevent an acute hemolytic reaction? A. Ensure that the client has a patent IV line before obtaining blood product from the refrigerator B. Obtain help from another nurse to confirm the correct client and blood product C. Take a complete set of vital signs before beginning transfusion and periodically during the transfusion D. Stay with the client for the first 15 to 30 minutes of the transfusion

B

A nurse is monitoring a client who takes aspirin 81 mg PO daily. The nurse should identify which of the following manifestations as adverse effects of daily aspirin therapy? (Select all that apply) A. Hypertension B. Coffee-ground emesis C. Tinnitus D. Paresthesias of the extremities E. Nausea

B, C, E

A nurse is monitoring a client who is receiving epoetin Alfa for adverse effects. The nurse should identify which of the following findings as an adverse effect of this medication? (Select all that apply) A. Leeukocytosis B. Hypertension C. Edema D. Blurred vision E. Headache

B, E

A nurse is planning to administer subcutaneous enoxaparin 40 mg using a refilled syringe of enoxaparin 40mg/0.4mL to an adult client following hip arthroplasty. Which of the following actions should the nurse plan to take? A. Expel the air bubble from the prefilled syringe before injecting. B. Insert the needle completely into the client's tissue. C. Administer the injection in the client's thigh. D. Aspirate carefully after inserting the needle into the client's skin.

B.

A nurse is creating a dietary plan for an adult female client who has a hemoglobin level of 9.8 g/dL. Which of the ff. foods should the nurse recommend? a. Carrots b. Raisins c. Maple Syrup d. Orange Juice

B. Foods high in iron are recommended to improve a low hemoglobin level. Raisins are a high source of iron. Foods high in iron are recommended to improve a low hemoglobin level. Carrots are not high in iron. Foods high in iron are recommended to improve a low hemoglobin level. Molasses, rather than maple syrup, is high in iron. Foods high in iron are recommended to improve a low hemoglobin level. Although orange juice is not high in iron, it does enhance the absorption of iron.

A nurse is creating a dietary plan for an adult female client who has a hemoglobin level of 9.8 g/dL. Which of the finding foods should the nurse recommend? a. Carrots b. Raisins c. Maple Syrup d. Orange Juice

B. Foods high in iron are recommended to improve a low hemoglobin level. Raisins are a high source of iron. -- Foods high in iron are recommended to improve a low hemoglobin level. Carrots are not high in iron. Foods high in iron are recommended to improve a low hemoglobin level. Molasses, rather than maple syrup, is high in iron. Foods high in iron are recommended to improve a low hemoglobin level. Although orange juice is not high in iron, it does enhance the absorption of iron.

A nurse is caring for a client who recently had surgery for insertion of a permanent pacemaker. Which of the ff. prescriptions should the nurse clarify? a. Serim cardiac enzyme level b. MRI of the chest c. physical therapy d. Low-Sodium diet

B. A permanent pacemaker is a contraindication for MRI of the chest. The magnets in the machine can create electromagnetic interference and cause the pacemaker to malfunction. A permanent pacemaker is not a contraindication for the measurement of cardiac enzymes. A permanent pacemaker is not a contraindication for physical therapy. A permanent pacemaker is not a contraindication for a low-sodium diet. Limiting sodium intake is a general recommendation for clients who have cardiovascular problems.

A nurse is assessing a client who is receiving a blood transfusion. Which of the ff. finding is a manifestation of hemolytic transfusion reaction? a. HTN b. Report of low-back pain c. Pallor d. Report of metallic taste

B. Low-back pain, fever, and chills are manifestations of a hemolytic transfusion reaction. The nurse should discontinue the transfusion and administer 0.9% sodium chloride through new IV tubing. Hypotension is a manifestation of a hemolytic transfusion reaction. Flushing and tachycardia are manifestations of a hemolytic transfusion reaction. Tachypnea and hemoglobinuria are manifestations of a hemolytic transfusion reaction.

A nurse is assessing a client who is receiving a blood transfusion. Which of the ff. finding is a manifestation of hemolytic transfusion reaction? a. HTN b. Report of low-back pain c. Pallor d. Report of metallic taste

B. Low-back pain, fever, and chills are manifestations of a hemolytic transfusion reaction. The nurse should discontinue the transfusion and administer 0.9% sodium chloride through new IV tubing. -- Hypotension is a manifestation of a hemolytic transfusion reaction. Flushing and tachycardia are manifestations of a hemolytic transfusion reaction. Tachypnea and hemoglobinuria are manifestations of a hemolytic transfusion reaction.

A nurse is providing discharge teaching to a client who has an implantable cardioverter/defribrillator (ICD). Which of the ff. information should the nurse include? a. the client cannot travel by air due to security screening b. the client should hold his cell phone on the side opposite the ICD c. the client should avoid the use of small electric devices. d. the client can carry his ICD in a small pocket.

B. The client should keep his cellular phone on the side opposite the ICD, as close proximity could interfere with the ICD's function. The client may travel by air but should inform security personnel of the presence of the ICD because metal detectors may sound an alarm. Small electrical devices do not affect the function of an ICD. The surgeon implants the ICD's generator under the client's skin in the left pectoral area.

A nurse is caring for who has esophageal varices and is hypotensive after vomiting 500mL of blood. Which of the following actions is the nurse's priority? a. Elevate the client's feet b. Increase the client's IV fluid rate c. Initiate a dopamine IV infusion for the client d. Administer a unit of packed RBCs.

B. When using the urgent vs. nonurgent approach to client care, the nurse should determine that the priority action is to increase the client's IV fluid rate. Providing fluid to the client will restore circulating volume and increase blood pressure. The nurse should elevate the client's feet to increase perfusion to the brain during the hypotensive episode, but this action is not the priority. The nurse might need to initiate a dopamine IV infusion to treat the client's hypotension, but this action is not the priority. The nurse should plan to administer a unit of packed RBCs to treat the hypotension and the blood loss, but this action is not the priority.

A nurse is caring for who has esophageal varices and is hypotensive after vomiting 500mL of blood. Which of the following actions is the nurse's priority? a. Elevate the client's feet b. Increase the client's IV fluid rate c. Initiate a dopamine IV infusion for the client d. Administer a unit of packed RBCs.

B. When using the urgent vs. nonurgent approach to client care, the nurse should determine that the priority action is to increase the client's IV fluid rate. Providing fluid to the client will restore circulating volume and increase blood pressure. -- The nurse should elevate the client's feet to increase perfusion to the brain during the hypotensive episode, but this action is not the priority. The nurse might need to initiate a dopamine IV infusion to treat the client's hypotension, but this action is not the priority. The nurse should plan to administer a unit of packed RBCs to treat the hypotension and the blood loss, but this action is not the priority.

Coagulation studies

BTT which contains Sodium Citrate

Increased number of basophils is called

Basophilia

RBC Descriptions: Blue dots seen in RBC on routine stain (lead)

Basophilic Stipling

Granulocytes include: AKA Granular Leukocyte (WBC) They HAVE granules in cytoplasm

Basophils (Rare in dog/cat/horse) Eosinophils (0-5% in dog/cat/horse) Neutrophils [SEGS] (55-85% dog/cat) (50-50% horse)

Why don't we quantify or list a cell as being Hyperchromic?

Because there can't be TOO MUCH hemoglobin in a cell Typically this will be an artifact (hemolytic sample)

A client is undergoing tests for multiple myeloma. Diagnostic study findings in multiple myeloma include:

Bence Jones protein in the urine.

A client is admitted to an acute care facility with esophageal cancer. The incidence of esophageal cancer is highest in:

Black males.

How is 1* Absolute Polycythemia treated?

Bleed the animal to lower the PCV and replace with fluids to thin out the remaining RBC's

PCV in conjunction with LTT

Blue Ring Crit Tube which contains no anticoagulant

A nurse is assessing a client during transfusion of a unit of whole blood. The client develops cough, shortness of breath, elevated blood pressure, and distended neck veins. The nurse should anticipate a prescription for which of the following medicatioons? A. Epinephrine B. Lorazepam C. Furosemide D. Diphenhydramine

C

A nurse is caring for a client who is receiving daily doses of oprelvekin. Which of the following lab values should the nurse monitor to determine effectiveness of this medication? A. Hemoglobin B. Absolute neutrophil count C. Platelet count D. Total white blood count

C

A nurse is caring for a hospitalized client who has an activated partial thrombophlebitis time (aPTT) greater than 1.5 times the expected reference range. Which of the following blood products should the nurse prepare to transfuse? A. Whole blood B. Platelets C. Fresh frozen plasma D. Packed red blood cells

C

A nurse is caring for a client who has atrial fibrillation and a new prescription for dabigatran to prevent development of thrombosis, Which of the following medications is prescribed concurrently to treat an adverse effect of dabigatran. A. Vitamin K1 B. Protamine C. Omeprazole D. Probenecid

C.

A nurse is caring for a hospitalized client who is receiving IV heparin for a deep-vein thrombosis. The client beings vomiting blood. After the heparin has been stopped, which of the following medications should the nurse prepare to administer? A. Vitamin K1 B. Atropine C. Protamine D. Calcium gluconate

C.

A nurse is caring for a client who has a prescription for one unit of packed RBCs. The nurse should plan to remain in the room with the client at which of the ff. times during the infusion to observe for a transfusion reaction? a. first 2min b. final 2min c. first 15min d. final 15min

C. The nurse should remain in the room during the first 15 min of the infusion, which is the most critical time period for monitoring a client for a transfusion reaction. Severe reactions usually occur during the infusion of the first 50 mL of blood.

A nurse is caring for a young female adult client who reports weakness, fatigue, and heavy menstrual periods. The client has a Hgb of 8 g/dL and a Hct of 28 g/dL. The nurse suspect which of the ff. types of anemia? a. Folic acid deficiency anemia b. Pernicious anemia c. Iron-deficiency anemia d. Sickle cell anemia

C. Iron-deficiency anemia results from poor gastrointestinal absorption of iron, a diet that is deficient in iron, or blood loss. The nurse should expect a client who has iron-deficiency anemia to have weakness, pallor, fatigue, reduced tolerance for activity, and cheilosis (ulcerations of the corners of the mouth). The nurse should expect a client who has folic acid deficiency to have pallor and jaundice; a smooth, beefy-red tongue (glossitis); fatigue; and weight loss. This type of anemia is caused by nutritional deficiencies, malabsorption syndromes (Crohn's disease), and medications (e.g., anticonvulsants, oral contraceptives). A client who has pernicious anemia is unable to absorb vitamin B12 due to a lack of intrinsic factors in the stomach. The nurse should expect this client to have pallor and jaundice; a smooth, beefy-red tongue (glossitis); fatigue; weight loss; and paresthesias to the hands and feet. Sickle cell anemia is an autosomal recessive disorder in which the RBCs develop a sickle shape following conditions in which decreased oxygen is available. These sickled cells then clump together and become fragile, causing tissue ischemia leading to eventual organ damage. Manifestations of sickle cel anemia include pain, pallor, cyanosis, dyspnea, fatigue, and weakness.

A nurse is caring for a young female adult client who reports weakness, fatigue, and heavy menstrual periods. The client has a Hgb of 8 g/dL and a Hct of 28 g/dL. The nurse suspect which of the ff. types of anemia? a. Folic acid deficiency anemia b. Pernicious anemia c. Iron-deficiency anemia d. Sickle cell anemia

C. Iron-deficiency anemia results from poor gastrointestinal absorption of iron, a diet that is deficient in iron, or blood loss. The nurse should expect a client who has iron-deficiency anemia to have weakness, pallor, fatigue, reduced tolerance for activity, and cheilosis (ulcerations of the corners of the mouth). -- The nurse should expect a client who has folic acid deficiency to have pallor and jaundice; a smooth, beefy-red tongue (glossitis); fatigue; and weight loss. This type of anemia is caused by nutritional deficiencies, malabsorption syndromes (Crohn's disease), and medications (e.g., anticonvulsants, oral contraceptives). A client who has pernicious anemia is unable to absorb vitamin B12 due to a lack of intrinsic factors in the stomach. The nurse should expect this client to have pallor and jaundice; a smooth, beefy-red tongue (glossitis); fatigue; weight loss; and paresthesias to the hands and feet. Sickle cell anemia is an autosomal recessive disorder in which the RBCs develop a sickle shape following conditions in which decreased oxygen is available. These sickled cells then clump together and become fragile, causing tissue ischemia leading to eventual organ damage. Manifestations of sickle cel anemia include pain, pallor, cyanosis, dyspnea, fatigue, and weakness.

A nurse is caring for a client with a ventricular pacemaker who is on ECG monitoring. The nurse understands that the pacemaker is functioning properly when which of the ff. appears on the monitor strip? a. Pacemaker spikes after each QRS complex b. Pacemaker spikes before each P wave c. Pacemaker spikes before each QRS complex d. Pacemaker spikes with each T wave

C. The pacemaker fires, showing a spike on the monitor strip, which stimulates the ventricle, and the QRS complex appears, indicating that depolarization has occurred. This indicates improper functioning. This is seen with an atrial pacemaker. This indicates improper functioning.

What is the Red Ring Hematocrit Tube used for? What anticoagulant is contained within a Red Ring Crit Tube ?

Can be used for small, sequential blood draws for PCV (packed cell volume), TP (total protein) and cell counts. Can be used to draw blood directly from a patient. Cannot be used for a blood film The anticoagulant Heparin

"A 45-year-old client receiving radiation therapy for thyroid cancer complains of mouth and throat pain. While inspecting the mouth and throat, the nurse notices white patches and ulcerations in the oral mucosa. What do these findings suggest?

Candidiasis

What are some advantages of EDTA anticoagulant?

Causes least amount of cell distortion/destruction, Doesn't interfere with staining of blood, Fairly stable for 4-6 hours, Best preservation of cells

A nurse is caring for a client who is postoperative following a urinary diversion to treat bladder cancer. Which of the following interventions should the nurse include in the plan of care? Empty the collection pouch when it is 2/3 full. Expect urine outflow into pouch to begin 1 to 2 days following surgery. Change the collection pouch in the early morning. Place an aspirin in the collection pouch to control odor.

Change the collection pouch in the early morning.

The nurse is teaching a group of women to perform breast self-examination. The nurse should explain that the purpose of performing the examination is to discover

Changes from previous self-examinations

What is the fluid portion of blood from a BTT (Blue Top Tube) called?

Citrated Plasma

healthcare professional is preparing to administer prescribed dose of filgrastim him to a patient the patient's laboratory reports indicate that the patient has a wbc count of 12,000 mm3 which of the action should the healthcare professional take

Clarify the prescription with the provider

"For a client with newly diagnosed cancer, the nurse assists in formulating a nursing diagnosis of Anxiety related to the threat of death secondary to cancer diagnosis. Which expected outcome would be appropriate for this client?

Client verbalizes feelings of anxiety.

For a client with newly diagnosed cancer, the nurse assists in formulating a nursing diagnosis of Anxiety related to the threat of death secondary to cancer diagnosis. Which expected outcome would be appropriate for this client?

Client verbalizes feelings of anxiety.

A client diagnosed with acute myelocytic leukemia (AML) has been receiving chemotherapy. During the last two cycles of chemotherapy, the client developed severe thrombocytopenia requiring multiple platelet transfusions. The client is now scheduled to receive a third cycle. How can the nurse best detect early signs and symptoms of thrombocytopenia?

Closely observe the client's skin for petechiae and bruising.

What error in collection of CBC completely invalidates the test?

Clots

"A client suspected of having colorectal cancer will require which diagnostic study to confirm the diagnosis?

Colonoscopy

What does CBC stand for?

Complete Blood Count - measurement of the size, number and maturity of the differenct blood cells (RBC's, WBC's & platelets) in a specific volume of blood, a CBC should also include a blood film review.

What is in the Diatomaceous Earth Tube? What does this detect?

Contains diatoms (little sea creatures) Stimulates the coagulation cascade & is indicated for determining activated clotting times (ACT)

The formula (math) for the Corrected WBC is -

Corrected WBC = nucleated RBC's x (100 / [NRBC's + 100]

A client with supraglottic cancer undergoes a partial laryngectomy. Postoperatively, a cuffed tracheostomy tube is in place. When removing secretions that pool above the cuff, the nurse should instruct the client to

Cough as the cuff is being deflated

T or F?

Counting punctates in a cat would artifactually increase the retic count?

Patient to begin therapy with recombinant factor IX to treat hemophilia B. the patient asked about the disease transmission. the healthcare professional should explain the recombinant factor IX eliminates the risk for what

Creutzfeldt-jakob disease

A client is undergoing a diagnostic workup for suspected testicular cancer. When obtaining the client's history, the nurse checks for known risk factors for this type of cancer. Testicular cancer has been linked to?

Cryptorchidism (failure of one or both testes to descend into the scrotum)

A nurse is caring for a client who is recievig a unit of packed red blood cells. Fifteen minutes following the start of the transfusion, the nurse notes that the client is febrile, with chills and red-tinged urine. Which of the following transfusion reactions should the nurse suspect? a. Febrile b. Allergic c. Acute Pain d. Hemolytic

D A hemolytic transfusion reaction occurs when antibodies in the recipient's blood react to foreign blood cells introduced by the transfusion. The antibodies bind to the foreign cells and destroy them in a process known as hemolysis. The destroyed cells are excreted by the kidneys (hemoglobinuria), causing the red-tinged urine. Hemolytic transfusion reactions can result in acute renal injury, disseminated intravascular coagulation, and circulatory collapse. -- A febrile transfusion reaction can occur in clients who have received multiple blood product transfusions. It is a response in which anti-white blood cell (WBC) antibodies react with the WBCs remaining in the blood product. This results in chills, fever, hypotension, tachycardia and tachypnea. Clients who have a history of multiple blood product transfusions may receive leukocyte reduced blood or single-donor HLA matched platelets along with a WBC filter to prevent febrile reactions. Allergic (anaphylactic) transfusion reactions occur most often in clients who have pre-existing allergies. It is thought to be the result of a reaction to the plasma protein contained in the blood product. Manifestations include urticaria, itching, and flushing. In extreme cases, bronchospasm and laryngeal edema, and shock may occur. Onset may occur as late as 24 hr following the transfusion. Clients who have a history of allergies may receive blood products in which the WBCs, plasma, and immunoglobulin A has been removed or the client may be pre-treated with antihistamines and corticosteroids. An acute pain transfusion reaction can occur during or following transfusion with blood products. It manifests as severe chest, joint, and back pain, along with hypertension and flushing of the face and neck. The client is often anxious. Acute pain transfusion reactions are treated symptomatically with medications for pain and rigors.

A nurse is preparing to administer filgrastim for the first time to a client who has just undergone a bone marrow transplant. Which of the following intervention is appropriate? A. Administer IM in a large muscle mass to prevent injury. B. Ensure that the medication is refrigerated until just prior to administration. C. Shake vial gently to mix well before withdrawing dose. D. Discard vial after removing one dose of the medication.

D.

A nurse is planning care for a client who is to receive packed RBCs. The nurse should plan for the total infusion time to not exceed which of the ff? a. 2hr b. 6hr c. 8hr d. 4hr

D. The nurse should infuse the packed RBCs for no longer than 4 hr due to temperature inconsistencies that develop over time and the possibility of bacterial contamination.

A nurse is teaching a client who has a new dx. of aplastic anemia. Which of the ff. information should the nurse include in the teaching? a. Aplastic anemia is associated with a decreased intake of iron. b. Aplastic anemia is associated with a decreased intake of iron. c. Aplastic anemia results in an inability to absorb vitamin B12. d. Aplastic anemia results from decreased bone marrow production of RBCs.

D. Aplastic anemia is a hypoproliferative anemia resulting from decreased production of RBC within the bone marrow. An inadequate intake of iron can result in iron deficiency anemia rather than aplastic anemia. Autoimmune hemolytic anemia, rather than aplastic anemia, is associated with an increased rate of RBC destruction. Pernicious anemia is seen in clients who lack the intrinsic factor responsible for vitamin B12 absorption.

A nurse is teaching a client who has a new dx. of aplastic anemia. Which of the ff. information should the nurse include in the teaching? a. Aplastic anemia is associated with a decreased intake of iron. b. Aplastic anemia is associated with a decreased intake of iron. c. Aplastic anemia results in an inability to absorb vitamin B12. d. Aplastic anemia results from decreased bone marrow production of RBCs.

D. Aplastic anemia is a hypoproliferative anemia resulting from decreased production of RBC within the bone marrow. -- An inadequate intake of iron can result in iron deficiency anemia rather than aplastic anemia. Autoimmune hemolytic anemia, rather than aplastic anemia, is associated with an increased rate of RBC destruction. Pernicious anemia is seen in clients who lack the intrinsic factor responsible for vitamin B12 absorption.

A nurse at a blood donation center is screening clients for blood donation. The nurse should identify that which of the ff. clients must reschedule donation? a. a client who weighs 50kg (110 lbs) and plans to donate 450mL of blood. b. A client who is 14yrs. of age c. A client who is Rh-positive d. A client who has an oral temperature of 37.8 or (100 F)

D. A client who has an oral temperature that exceeds 37.5° C (99.6° F) defers eligibility to donate blood. The client should weigh at least 50 kg (110 lb) to donate 450 mL of blood. The client who is 14 years of age is not eligible to donate blood: however, a client who is 16 or 17 years of age with a parental consent is eligible to donate blood. Blood type does not affect eligibility to donate blood.

A nurse at a blood donation center is screening clients for blood donation. The nurse should identify that which of the ff. clients must reschedule donation? a. a client who weighs 50kg (110 lbs) and plans to donate 450mL of blood. b. A client who is 14yrs. of age c. A client who is Rh-positive d. A client who has an oral temperature of 37.8 or (100 F)

D. A client who has an oral temperature that exceeds 37.5° C (99.6° F) defers eligibility to donate blood. -- The client should weigh at least 50 kg (110 lb) to donate 450 mL of blood. The client who is 14 years of age is not eligible to donate blood: however, a client who is 16 or 17 years of age with a parental consent is eligible to donate blood. Blood type does not affect eligibility to donate blood.

A nurse received a unit of packed RBCs from a blood bank and notes that the time is 1130. The nurse should begin the infusion at which of the ff. times? a. when the client has finished eating lunch b. when the client states he is ready to start the infusion c. 2 hr after obtaining blood from the blood bank d. As soon as the nurse can prepare the client and the administration set.

D. The nurse should infuse the blood as soon as possible and complete the procedure within 4 hr. The nurse should begin the infusion as soon as possible after obtaining the packed RBCs from the blood bank. This delay is unnecessary and can complicate the requirement to complete the infusion within 4 hr. This delay is unnecessary and increases the risk of bacterial contamination since the blood is out of the refrigerator. The nurse should infuse the blood as soon as possible after obtaining the packed RBCs from the blood bank.

A nurse is preparing to administer dabigatran to a client who has atrial fibrillation. The nurse should explain that the purpose of this medication is which of the following? a. to convert atrial fibrillation to sinus rhythm b. to dissolve clots in the bloodstream c. to slow the response of the ventricles to the fast atrial impulses d. to reduce the risk of stroke in clients who have atrial fibrillation.

D. Clients who have atrial fibrillation are at an increased risk for thrombus formation and subsequent embolization to the brain. Anticoagulants, such as dabigatran, help prevent thrombus formation. Antidysrhythmic medications such as diltiazem are used to slow the ventricular rate for clients who have atrial fibrillation. Thrombolytic medications, such as alteplase, are used to remove thrombi that have already formed. Beta-blockers such as carvedilol slow the heart rate.

A nurse received a unit of packed RBCs from a blood bank and notes that the time is 1130. The nurse should begin the infusion at which of the ff. times? a. when the client has finished eating lunch b. when the client states he is ready to start the infusion c. 2 hr after obtaining blood from the blood bank d. As soon as the nurse can prepare the client and the administration set.

D. The nurse should infuse the blood as soon as possible and complete the procedure within 4 hr. -- The nurse should begin the infusion as soon as possible after obtaining the packed RBCs from the blood bank. This delay is unnecessary and can complicate the requirement to complete the infusion within 4 hr. This delay is unnecessary and increases the risk of bacterial contamination since the blood is out of the refrigerator. The nurse should infuse the blood as soon as possible after obtaining the packed RBCs from the blood bank.

Patient is taking ferrous sulfate to treat iron deficiency anemia and develop severe nausea and vomiting what should be used to treat the complication

Deferoxamine

Abnormally shaped / Unequal sized RBC's:

Definition of Anesocytosis

What is the mechanism or "direct cause" for relative polycythemia (AKA increased RBC concentration)?

Dehydration (same # of RBC's contained in less fluid) Splenic Contraction which releases RBC's (race horse, prey animal who has to run to be safe)

How can breast cancer prevention programs best serve women who are at risk and come from lower socioeconomic backgrounds?

Develop screening and educational programs.

Platelets are...

Disk shaped, non-nucleated cells which adhere to uneven or damaged surfaces. Largest Bone Marrow Cell - 1/3 in spleen, 2/3 in circulating blood

What fluid is used to calibrate & clean the machine used to measure plasma proteins? What is the specific gravity of this fluid?

Distilled H2O & its specific gravity is 1.000

Specific gravity has no measurement unit. Reported out to the thousandth place.

Distilled H2O has a specific gravity of 1.000. Distilled H2O is what is used to calibrate the refractometer.

WBC (white blood cell) Identification: Also called......LEUKOCYTES

Do the cells appear (do I see) granules? If yes - what are the granule colors? Pink - Eosinopil Blue/Blue-Grey - Basophil Dusty Purple (do you have a cow sample?) - Yes - then it is a cow seg If you do not see granules - Is there a dark, dense, segmented nucleus? Yes? Seg No? what is the nucleus to cytosplasm ratio? High: Lymphocyte (nucleas round/oval?) (chromatin smudged?) Low: Monocyte (nucleus ameboid w/lacy chromatin? Vacules?)

"A client seeks care for hoarseness that has lasted for 1 month. To elicit the most appropriate information about this problem, the nurse should ask which question?

Do you smoke cigarettes, cigars, or a pipe?

A client who has a suspected ovarian tumor is scheduled to have a pelvic ultrasound. In preping the client for the procedure, the nurse instructs the client to- a, drink 4 to 6 glasses of water and do not void b. have nothing to eat or drink after midnight c. self-administer an enema the night before the procedure d. abstain from sexual inercourse the night before

Drink 4 to 6 glasses of water and do not void. a pelvic ultrasound requires a full bladder to better visualize and identify the organs and structures in the pelvis.

A client who has a suspected ovarian tumor is scheduled to have a pelvic ultrasound. In preparing the client for the procedure, the nurse should reinforce to the client to

Drink 4-6 glasses of water and do not void

A client is receiving chemotherapy to treat breast cancer. Which data collection finding indicates a fluid and electrolyte imbalance induced by chemotherapy?

Dry oral mucous membranes and cracked lips

When hypoxia occurs

EPO (hormone responsible for RBC formation) is made in response from the kidneys

A patient is to begin taking ferrous sulfate to treat iron deficiency anemia when taking the drug what should be included in the following instructions

Eat iron enriched foods spread the dosage across each day take the drug on an empty stomach increased dietary fiber

RBC Descriptions: Equally shaped projections occur on RBC

Echinocyte

Patient has mild hemophilia A and is to begin taking desmopressin (DDAVP) to prevent bleeding what should they monitor for adverse reaction

Edema

A client is being evaluated in an oncology clinic after referral by the client's provider for suspicion of Hodgkin's disease. The nurse should anticipate that the client will exhibit which of the following?

Enlarged lymph nodes

A nurse in an oncology clinic is assessing a client who has early stage Hodgkin's lymphoma. Which of the following findings should the nurse expect? Bone and joint pain Enlarged lymph nodes Intermittent hematuria Productive cough

Enlarged lymph nodes · Hodgkin's lymphoma is a malignancy of lymphoid tissue found in the lymph nodes, spleen, liver, and bone marrow. The first manifestation of this cancer is often an enlarged painless lymph node, or nodes, which appear without a known cause. Other early manifestations include night sweats, unexplained weight loss, fever, and pruritus. The disease can spread to adjacent lymph nodes and later might spread outside the lymph nodes to the lungs, liver, bones, or bone marrow. The spread of Hodgkin's lymphoma is usually in an ordered pattern.

Decreased eosinophils is called

Eosinopenia

Increased eosinophils is called

Eosinophilia

How do you make a reticulocyte blood film?

Equal parts EDTA whole blood and NMB Stain. Incubate 10-20 minutes (no longer than 30) @ room temp. Make a very thin blood film then count retics on 100X. Count 1000 RBC's and tally retics while counting.

EPO is.... What are 3 effects of EPO?

Erythropoetin is a glycoprotein hormone that controls erythropoiesis, or red blood cell production. 1. Stimulates Pluiripotential Stem Cells (PPSC) to differentiate into rubriblasts (1st stage of RBC maturation) 2. Increases hemoglobin synthesis 3. Early release of nucleated RBC's (polychromatophilic cells) into peripheral blood - because the need for RBC's is so dire.

If you are evaluating WBC'S it can also be called...

Evaluating the LEUKOGRAM which consists of looking at the total WBC count & distribution of the granulocytes and a-granulocytes

The client is receiving an infusion of cytarabine (Cytosar-U) through a peripheral I.V. catheter when he complains of burning at the insertion site. The nurse notes no blood return from the catheter, but she sees redness at the I.V. site. The client is most likely experiencing which complication?

Extravasation

An animal with kidney failure will have a macrocytic hypochromic regenerative anemia

FALSE

Basophilic stippling is associated with oxidizing substances like onion & Tylenol

FALSE

In early gestastion an animal will have an absolute proteinemia

FALSE

Increased consumption of proteins will cause an absolute hyperproteinemia

FALSE

Metarubricytes have a kidney bean shaped nucleus

FALSE

Monocytes circulate for 10 hours then go into tissue

FALSE

A nurse is helping to develop a plan of care for a client with metastatic lung cancer who is at risk for the development of superior vena cava syndrome. The nurse includes in the plan to assess the client frequently for early manifestations of superior vena cava syndrome including

Facial edema

A nurse is caring for a client who has lung cancer that has metastasized. Which of the following findings indicates the client is developing superior vena cava syndrome? Irregular cardiac rhythm Numbness in the hands Muscle cramps Facial edema

Facial edema · Superior vena cava syndrome is a medical emergency resulting from a partial occlusion of the superior vena cava, leading to a decreased blood flow through the vein. Most cases of superior vena cava syndrome are associated with cancers involving the client's upper chest, such as advanced lung and breast cancers and lymphoma. The earliest manifestations of superior vena cava syndrome are facial and upper extremity edema. Death can result if the compression is not corrected.

A nurse is developing a plan of care for a client with metastatic lung cancer who is at risk for the development of superior vena cava syndrome. The nurse includes in the plan to assess the client frequently for early manifestations of the superior vena cava syndrome including- a. irregular cardiac rhythm b. change in the level of consciousness c. wasting of the arms d. facial edema

Facial edema. superior vena cava syndrome is a partial occlusion of the superior vena cava. lower than normal blood flow through this major vein which causes facial and upper extremity edema.

A reticulocyte count is indicated to determine the bone marrow's response to anemia in the horse.

False - The horse will not release polychromatophilic cells early

A client with cancer is scheduled for radiation therapy. The nurse knows that radiation at any treatment site may cause a certain adverse effect. Therefore, the nurse should prepare the client to expec

Fatigue

Fibrinogen is

Fibrinogen is a protein that plays a key role in blood clotting. Fibrinogen is a sticky, fibrous coagulant in the blood. Fibrinogen is produced by the liver. This protein helps stop bleeding by helping blood clots to form.

In ruminants in states of inflammation

Fibrinogen will increase prior to WBC's. Fibrinogen test will be an initial clue that inflammation is going on.

Patient is undergoing bone marrow transplantation following a high dose of chemotherapy what drugs should they expect to help the patient's risk of infection

Filgrastim (neupogen)

A patient is known to have a sensitivity to E. coli derived proteins the sensitivity is a contraindication of what drug

Filgrastim neupogen patients who are sensitive to E.coli-derived proteins or have acute respiratory distress syndrome should not receive filgrastim, leukopoietic growth factor.

The nurse is teaching a client who suspects that she has a lump in her breast. The nurse instructs the client that a diagnosis of breast cancer is confirmed by:

Fine needle aspiration and biopsy.

A nurse is caring for a client who has just begun chemo for acute lymphoma. The plan of care indicates that signs of tumor lysis syndrome should be part of the nursing assessment. What asssessment finding is consistent with tumor lysis syndrome? a. polyuria b. muscle weakness c. flank pain d. hypotension

Flank Pain: Tumor lysis syndrome is caused by the sudden, rapid death of cells, particularly cancer cells in clients with leukemia or lymphoma. lab results will show high potassium, uric acid, and phosphorous levels and low calcium levels. progression of the syndrome may cause acute kidney failure, cardiac arrhythmias, seizures, loss of muscle control, and death. flank pain is a symptom of tumor lysis syndrome secondary to the strain placed on the kidneys to excrete large amounts of intracellular metabolites.

What is ROULEAUX? Think Elmer's Glue

Found in the body of a blood film it is where the red blood cells (RBC's) chain together

What is AGGLUTINATION? Think Super Glue

Found in the body of a blood film this is where the red blood cells (RBC's) have abnormal clustering or binding of the cells upon themselves. This indicates the antibodies are attacking themselves. Antigen/Antibody binding.

A staff nurse on the oncology unit must teach a new unit assistant about infection control practices on the unit. The nurse should explain that the most important measure to prevent the spread of infection is what?

Frequent hand washing

Total Protein functions & origins:

Functions: protein, enzymes, hormones, clotting (coagulation), defense (antibodies), structural matrix of a cell, transporter/carrier molecules, osmotic pressure in vasculature Origins: Liver & Lymph Tissue predominately

Hyperglycemia is caused by....

Glucose, is a major source of energy for the body, being overproduced or not kept under control by the pancreas. Insulin, a hormone that is produced and released by the pancreas into the bloodstream when glucose levels rise, plays a key role in maintaining normal sugar levels. Low levels or absolute deficiency of insulin results in abnormally high blood sugar levels.

Four weeks after a bone marrow transplant, a client develops a fever and a rash on the hands and feet. Based on these findings, the nurse should suspect that the client is experiencing which of the following conditions?

Graft-versus-host disease

4 weeks after a bone marrow transplant, a client develops a fever and a rash on the hands and feet. Based on theses findings, the nurse should suspect that the client is experiencing what condition? a. allergic response to medication b. side effects of radiation therapy c. veno-occlusive disease of liver d. graft-verses-host disease

Graft-versus-host disease. usually occurs between 10 and 100 days following the transplant. In GVHD, T-cells from the donor bone marrow attack and destroy vulnerable host cells. glucocorticoids and cyclosporine may be used to treat the condition.

Blood Gases

Green TT which contains Heparin

Blood Glucose

Grey TT which contains Oxylate

A nurse is caring for a client who has been diagnosed with cancer in situ. The clients asks the nurse what type of cancer this is. An appropriate response by the nurse would be that cancer in situ is when abnormal cell production

Has developed within a localized area

A nurse is caring for a client who has been diagnosed with cancer in situ. The client asks the nurse what type of cancer this is. The nurse should respond that cancer in situ is when abnormal cell production-- a. has spread to a distant site b. has infiltrated the lymph nodes c. has invaded surrounding tissue d. has developed within a localized area

Has developed within a localized area. cancer-in-situ is an early stage of cancer that is limited to the site of origin.

What should a male client older than age 50 do to help ensure early identification of prostate cancer?

Have a digital rectal examination and prostate-specific antigen (PSA) test done yearly.

The nurse is speaking to a group of women about early detection of breast cancer. The average age of the women in the group is 47. Following the American Cancer Society guidelines, the nurse should recommend that the women:

Have a mammogram annually.

Patient is to begin factor VIII therapy to treat hemophilia when administering it what action should the healthcare professional take

Have emergency equipment ready

Which of the following actions displayed by a grieving husband over his dying wife would cause the nurse to suggest counseling?

He refuses to acknowledge his wife's family and blames them for her current health problems.

Patient is to begin taking alteplase therapy what should patient monitor for adverse effects

Headache

Clear to pale projections off RBC on routine stain, pale blue in NMB (New Methyline Blue)

Heinz Bodies

T or F?

Hemolysis will invalidate the CBC?

TRUE

Hemolysis, increased BUN, increased glucose, and lipemia can all cause artifactual hyperproteinemia

List the abnormal colors for plasma, the proper term for each & one cause:

Hemolytic (red/pink) - IV Hemolysis Icteric (yellow) - Biliary Stasis Lipemic (white) - Diabetes Mellitus

A nurse is admitting a client who has been diagnosed as having squamous cell carcinoma of the lung. When completing the history and physical, the nurse should ask the client about a history of which of the following?

Hemoptysis

TRUE

Hemorrhage will cause an absolute decrease in PCV and absolute hypoproteinemia

There are 3 Mechanisms of Anemia 1. Decreased production of RBC's 2. Increased loss of RBC's 3. Increased destruction (from inside patient)

Here are examples of each of those mechanisms: 1. Bone Marrow Toxin, Kidney Failure, Iron Deficiency 2. Hemorrhage (GI Bleed) 3. IMHA - Immune Mediated Hemolytic Anemia

Patient is to begin taking important Epoetin Alfa what laboratory values would increase with therapy

Hgb Epoetin Alfa, an erythropoietic growth factor, increases the production of RBCs for pt who have anemia due to chronic renal failure or chemotherapy. Hgb and Hct should increase with effective therapy.

Hyperkalemia...

Higher than normal potassium levels in the blood

RBC Descriptions: Bizarre shaped RBC's, shape altered drastically

Howell Jolly Bodies

RBC Descriptions: Nuclear remnant

Howell Jolly Body

A client, age 42, visits the gynecologist. After examining her, the physician suspects cervical cancer. The nurse reviews the client's history for risk factors for this disease. Which history finding is a risk factor for cervical cancer?

Human papillomavirus infection at age 32

A home care nurse assesses for disease complications in a client with bone cancer. The nurse knows that bone cancer may cause which electrolyte disturbance?

Hypercalcemia

What is the term used to describe a higher than normal range for TP (Total Protein)

Hyperproteinemia

Epoetin Alfa is administered subcutaneously to a patient who has renal failure the healthcare professionals to monitor for what adverse affect

Hypertension

RBC Descriptions: RBC has less Hb (hemoglobin) than it should have

Hypochromic

What is the term used to describe a lower than normal range for TP (Total Protein)

Hypoproteinemia

The stimulus for erythropoiesis is... What hormone is produced? By what organ? This hormone has 3 effects on the bone marrow, what are they?

Hypoxia EPO Kidneys 1. will cause PPSC to differentiate into rubriblast 2. increase hemoglobin synthesis 3. Early release of RBC's into peripheral blood

An oncology clinic nurse is reinforcing prevention measures for oropharyngeal infections to a client receiving chemotherapy. Which statement by the client indicates an understanding of the nurse's teaching?

I clean my teeth gently several times a day.

A client is receiving external radiation therapy. Which of the following statements indicates to the nurse that the client understands the discharge teaching plan?

I need to protect the area from sunlight

A client is receiving external radiation therapy. What statement indicates to the nurse that the client understands the discharge teaching plan? a. I need to protect the area from sunlight b. I'm going to apply skin cream every day to the area c. I'll massage the area once a day d. I'll wash the markings off after each therapy treatment

I need to protect the area from sunlight.

A client becomes neutropenic 11 days after his last chemotherapy cycle. It's obvious that the client understands his condition when he states:

I'll monitor my temperature frequently and go to the nearest emergency department if my temperature rises above 100.4° F."

The nurse is caring for a recently married, 29-year-old female client, who was diagnosed with acute lymphocytic leukemia (ALL). The client is preparing for an allogeneic bone marrow transplant. Which statement by the client demonstrates an understanding of the physician's description of the diagnosis and treatment?

I'll only need chemotherapy therapy before receiving my bone marrow transplant.

"A client receiving chemotherapy has a nursing diagnosis of Deficient diversional activity related to decreased energy. Which statement by the client indicates an understanding of appropriate ways to deal with this deficit?

I'll play card games with my friends.

Which statement by a client undergoing external radiation therapy indicates the need for further teaching?

I'm afraid I'll expose my family members to radiation.

"A client recently diagnosed with metastatic liver cancer is admitted for hospice care. An acquaintance of the client calls the nurse to ask how the client is doing. Which response by the nurse is most appropriate?

I'm not permitted to give you any information about his condition for confidentiality reasons; I'll tell him you were asking about him.

Patient is to begin taking warfarin and what should the healthcare professional caution about with the over-the-counter drugs

Ibuprofen NSAIDS can increase the risk of bleeding.

The nurse is instructing a premenopausal woman about breast self-examination. The nurse should tell the client to do her self-examination:

Immediately after her menses

What does pregnancy do to the TP (Total Protein) levels of the mother?

In early gestation there is Hyperproteinemia due to immunoglobulins. In last 48 hours before birth mother will suffer from Hypoproteinemia because her hormones are being transferred into the colostrum to help protect the newborn.

T of F?

In horses we count just aggregates?

Reticulocyte production should

Increase in response to any loss of red blood cells - The disease process would be called anemia if this does not happen

If a blood film is too thin how could it be corrected?

Increase the angle of the spreader slide

FALSEenolu

Increased destruction is a mechanism of anemia and bone marrow toxins are an example of that

Primary Absolute Polycythemia (AKA Polycythemia Vera) Primary is Abbreviated as 1*

Increased production of RBC's caused by a red blood cell tumor where red blood cells become malignant and proliferate uncontrollably outside of the control of the body. These tumors are like teenagers they don't listen to anyone!!!

What use is there for the Lavendar (Purple) Top Tube? LTT or PTT What anticoagulant is contained in the LTT or PTT?

Indicated for CBC (complete blood count) studies Contains the anticoagulant EDTA which binds with calcium to prevent clotting (coagulation)

What would a technician use a Green Top Tube (GTT) for? What anticoagulant is contained in this tube?

Indicated for blood gas testing - CO2 and O2 levels Heparin is in a GTT, this chemical prevents conversion of prothrombin to thrombin

What would the use be for a blood sample that was taken in a Blue Top Tube (BTT)? What anticoagulant is contained in a BTT?

Indicated for coagulation studies Sodium Citrate which binds with calcium to prevent coagulation

A RED TOP TUBE is used for what? What does this tube contain? What other name is it called?

Indicated for use in blood chemistry studies This tube contains no chemical agent so blood WILL clot A serum separator tube

A client with a nagging cough makes an appointment to see the physician after reading that this symptom is one of the warning signs of cancer. What is another warning sign of cancer?

Indigestion

The nurse is caring for a client with bronchogenic carcinoma. Which nursing diagnosis takes highest priority?

Ineffective airway clearance related to obstruction by a tumor or secretions

Why does TP (Total Protein) get evaluated? What can this test help diagnose?

Inflammation, Immune Status, Liver Function (predominately Albumin), Kidney Function, GI Function (are we consuming & are we absorbing), Hydration

A client with metastatic brain cancer is admitted to the oncology floor. According to the Patient Self-Determination Act of 1991 (PSDA), what is the hospital required to do concerning the execution of advance directives?

Inform the client or legal guardian of his right to execute an advance directive.

Several days before admission, a client reports finding a small lump in the left breast near the nipple. What should the nurse tell the client to do?

Inform the physician immediately.

Patient is to begin taking aspirin to reduce the risk of cardiovascular event what should they understand and are the drug inhibits platelet aggregation by what mechanism

Inhibiting cyclooxygenase action and platelets

Epoetin alfa subcutaneously given to a patient who has renal failure the healthcare professional should take what following action

Insert the needle into the vile once

"For a client newly diagnosed with radiation-induced thrombocytopenia, the nurse should include which intervention in the plan of care?

Inspecting the skin for petechiae once every shift

What is one looking for in the monolayer of a blood film @ 10X?

Is Cell size appropriate & WBC distribution

Why does an "end of life" animal have hyperproteinemia?

It is relative (not a real value) because elderly animals have a more difficult time/tissues loose ability to hold H2O

Potassium is significant because...

It is so important to heart function. Potassium is both an electrolyte and a mineral. It helps keep the water (the amount of fluid inside and outside the body's cells) and electrolyte balance of the body. Potassium is also important in how nerves and muscles work. Potassium levels often change with sodium levels. When sodium levels go up, potassium levels go down, and when sodium levels go down, potassium levels go up. Potassium levels are also affected by a hormone called aldosterone, which is made by the adrenal glands.

What does it mean if a cell is Macrocytic?

It means the cells are larger than normal and usually these cells are younger, less mature RBC's being kicked out into the peripheral blood early (released early). a larger than normal red blood cell.

What does it mean if a cell is Microcytic?

It means the cells being released are small cells, not holding as much hemoglobin, In microcytic anemia, the red blood cells (erythrocytes) are usually also hypochromic

TRUE

It takes 5-7 days to see a response to anemia in the peripheral blood

Horse venipuncture sites & needle sizes

Jugular (preferred) - 18 ga (green)

Cattle venipuncture sites & needle sizes

Jugular (preferred) - 18 ga (green) Tail Vein - 20 ga (pink)

Sheep/Goats venipuncture sites & needle sizes

Jugular (preferred) - 20 ga (pink)

Dog venipuncture sites and needle sizes used for those sites:

Jugular (preferred) - 20 ga (pink) Cephalic - 22 ga (blue) Lateral Saphenous - 25 ga (red)

Cat/Cat sized dog venipuncture sites & needle sizes:

Jugular (preferred) - 22 ga (blue) Cephalic - 25 ga (red) Femoral - 25 ga (red)

Avians venipuncture sites & needle sizes

Jugular - <25 ga (black) Medial Metatarsal (leg)- <25 ga (black) Cutaneous Ulnar (wing) - <25 ga (black)

What are some disadvantages of using Oxylate as an anticoagulant? (Found in Gray Top Tube)

K oxylate shrinks cells NH4 oxylate causes cells to swell Cannot be used for BUN (blood-urea-nitrogen) tests due to the NH4 Oxylate crystals can be seen on differential so not able to use for CBC

"A client undergoes a laryngectomy to treat laryngeal cancer. When teaching the client how to care for the neck stoma, the nurse should include which instruction?

Keep the stoma moist.

RBC Descriptions: RBC's look like helmets

Keratocytes

"A client who underwent surgery 2 days ago is unable to tolerate anything by mouth and is experiencing mild to moderate cancer pain. Which nonsteroidal anti-inflammatory drug (NSAID) can the nurse safely administer by the I.M. route?

Ketorolac (Toradol)

For a fibrinogen test what supplies would you need?

LTT (Lavendar Top Tube) & 2 Blue Ring Crit Tubes

CBC

LTT which contains EDTA

RBC Descriptions: RBC has an increased membrane to volume ratio (FULL)

Leptocyte

When WBC count is elevated or increased that is called...

Leukocytosis

When WBC count is low that is called

Leukopenia or Leukocytopenia

"A client with cancer is being evaluated for possible metastasis. Which of the following is a common metastasis site for cancer cells?

Liver

Where are macrophage cells located? HINT: in what 3 organs?

Liver, spleen & bone marrow

Hypokalemia....

Lower than normal potassium levels in the blood

"A client with uterine cancer asks the nurse, ""Which type of cancer causes the most deaths in women?"" How should the nurse respond?

Lung cancer.

A client is undergoing a left modified radical mastectomy for breast cancer. Postoperatively, blood pressure should be obtained from the client's right arm, and the left arm and hand should be elevated as much as possible to prevent which condition?

Lymphedema

A-Granulocytes include: AKA Non-Granular Leukocyte (WBC) They have NO granules in cytoplasm

Lymphocytes (dogs/cats) 15-45% (horse) 50-50% Monocytes (dogs/cats/horse) 0-5%

Increased numbers of lymphs is called

Lymphocytosis

Decreased numbers of lymphs is called

Lymphopenia

Indices readings: Polychromatophilic (premature RBC's) Present

Macrocytic Hypochromic Regenerative Anemia

Aged or damaged RBC's are removed from the body primarily by what type of cell? What system is this called?

Macrophage cells Extravascular hemolysis

A client who reports increasing difficulty swallowing, weight loss, and fatigue is diagnosed with esophageal cancer. Because this client has difficulty swallowing, the nurse should assign highest priority to:

Maintaining a patent airway.

What is an advantage of using Oxylate as an anticoagulant? (Found in Gray Top Tube)

Maintains cell wall integrity eliminating movement of glucose from plasma into the cell

On a visit to the gynecologist, a client complains of urinary frequency, pelvic discomfort, and weight loss. After a complete physical examination, blood studies, and a pelvic examination with a Papanicolaou test, the physician diagnoses stage IV ovarian cancer. The nurse expects to prepare the client for which initial treatment?

Major surgery

MCH = The third test evaluated and included for checking anemia status but is really meant to double check (back up) the MCHC indice test evaluation

Mean Corpuscular Hemoglobin (Hb) = Hb X 10/RBC's - round to tenths place and (pg) is unit. MCV, MCHC, MCH are all mathematical indices used to classify anemia based on published ranges. These can help describe the RBC population.

Fibrinogen normal values for a dog are: Fibrinogen normal values for a cat are:

Measured in whole #'s in mg/dl. There are no listed normal values for a dog or a cat Fibrinogen levels are used to determine if the patient has inflammation

An oncology nurse educator is speaking to a women's group about breast cancer. Questions and comments from the audience reveal a misunderstanding of some aspects of the disease. Various members of the audience have made all of the following statements. Which statement is accurate?

Men can develop breast cancer.

RBC Descriptions: Nucleated RBC

Metarubricyte

Indices readings: Iron Deficiency

Microcytic Hypochromic Non-Regenerative Anemia

List what is evaluated in the Feathered Edge of a blood film @ 10X:

Microfilariae, Platelet Clumping, Abnormal Cells & RBC Morphology

Which drug should be prescribed with nonsteroidal inflammatory drugs (NSAIDs) in elderly cancer clients to reduce the risk of GI adverse effects?

Misoprostol (Cytotec)

T or F?

Mixing for an extended period of time or squirting blood into the tube can cause hemolysis?

FALSE

Monocytes and eosinophils will affect the total WBC count the most

Decreased numbers of monocytes is called

Monocytopenia

Increased numbers of monocytes is called

Monocytosis

This primary method for RBC removal from the blood stream is called....

Mononuclear Phagocytic System (MPS)

A sibling of a client recently diagnosed with colon cancer questions the nurse in regard to the CEA blood test. Info the nurse should include: "The CEA test is: a. most helpful in monitoring progress of the disease in clients already being treated for colon cancer. b. an effective screening test and is indicated because of family history c. recommended by the american cancer association to be performed yearly starting at age 50 d. used to confirm the diagnosis if a client has symptoms consistent with colon or rectal cancer

Most helpful in monitoring progress of the disease in clients already being treated for colon cancer. CEA is a substance produced by the cels of most colon and rectal cancers and released into the blood stream. CEA levels should return to normal following successful treatment. It is not used as a screening test for colon cancer in clients who have not been diagnosed with cancer and appear to be healthy.

A nurse is caring for a client who is to receive brachytherapy via a cervical implant. Which of the following statements by the client should indicate to the nurse that further teaching is necessary?

My children will only be allowed to visit for 20 minutes

Decreased neutrophils is called

Neutropenia

Increased neutrophils is called

Neutrophilia

A client who has leukemia has a relapse, and the provider recommends a bone marrow transplant. After the provider leaves, the client asks the nurse, "Do they put the marrow back in me the same way they took it out for a biopsy?" Which of the following is an appropriate response by the nurse?

No, it's transfused just like any other blood component into your central IV line

A client with leukemia suffers a relapse, and the provider recommends a bone marrow transplant. After the provider leaves, the client asks the nurse, "Do they put the marrow back in me the same way they took it out for a biopsy?" What response should the nurse make? a. no, it's transfused just like any other blood component into your central IV line b. yes, it's replaced directly into the bone like a biopsy, but it requires several sites c. yes, however, you will not feel it because they will do it in the operating room under anesthesia d. I'll get the dr back in here to answer the rest of your questions.

No, it's transfused just like any other blood component into your central IV line.

A client who is receiving external radiation asks the nurse, "Am I radioactive?" Which of the following is an appropriate response by the nurse?

No, you are not radioactive

According to the World Health Organization's analgesic ladder, which drugs would likely be used first to treat mild pain?

Nonsteroidal anti-inflammatory drugs (NSAIDs)

Indices readings: Bone Marrow Toxin

Normocytic Normochromic Non-Regenerative

Indices readings: Kidney Failure

Normocytic Normochromic Non-Regenerative Anemia

Indices readings: New Anemia

Normocytic Normochromic Non-Regenerative Anemia

Artifactual Hyperproteinemia can be caused by...

Not calibrating refractometer, lipemia (the presence of an excess of fats or lipids in the blood), increased BUN, increased glucose, hemolysis

Anemia is (defined as):

Not enough production of RBC's, or blood that as a lower than normal # of RBC's. Anemia can also occur if the RBC's don't contain enough hemoglobin (Hb)

What is the BIGGEST error in hematology?

Not mixing the sample - be sure and mix gently and thoroughly with the anticoagulants

T or F?

Number one error in hematology is hemolysis

At a health fair, an oncology nurse answers questions about risk factors for various types of cancer. One person with a family history of colon cancer asks the nurse to identify risk factors for this type of cancer. The nurse should identify

Obesity

"Which intervention is appropriate for the nurse caring for a client in severe pain receiving a continuous I.V. infusion of morphine?

Obtaining baseline vital signs before administering the first dose

"A 30-year-old client whose mother died of breast cancer at age 44 and whose sister has ovarian cancer is concerned about developing cancer. A nurse who is a member of the oncology multidisciplinary team should suggest that the client ask the physician about which of the following actions?

Obtaining genetic counseling

T or F?

One of the mechanisms by which effete (damaged) RBC's are removed is IVH (intra vascular hemolysis)?

A nurse is collecting a health history from a female client who is undergoing screening for breast cancer. Which of the following factors should the nurse identify for placing the client at the greatest risk for developing breast cancer? Obesity Oral contraceptive use Alcohol use Over 50 years of age

Over 50 years of age

A nurse is collecting a health history from a client. Which of the following findings is the highest risk factor for the client developing skin cancer? Age over 60 Genetic predisposition Light-skinned race Overexposure to sun light

Overexposure to sun light

Name the three parts of the hematocrit tube (from bottom to top) once spun and indicate what is contained in each portion:

Packed Clay Plug, 1. RBC's (contain the spun red blood cells) this is where the packed cell volume reading comes from 2. Buffy Coat (consists of platelets, WBC's (white blood cells), +/- microfilariae, and +/- nucleated red blood cells (NRBC's) 3. Plasma

Blood volume (PCV) is also known as?

Packed cell volume (PCV) % quantity of whole blood made up of RBC's (AKA Hematocrit levels)

Which finding is an early indicator of bladder cancer?

Painless, intermittent hematuria

A nurse is caring for a client who is receiving chemotherapy to treat cancer. Which of the following adverse effects should the nurse anticipate from the chemotherapy? Gingival hyperplasia Hirsutism Pancytopenia Weight gain

Pancytopenia · Pancytopenia, a deficiency of WBCs, RBCs, and platelet counts, is an expected adverse effect of chemotherapy.

A client is undergoing a diagnostic workup for suspected thyroid cancer. What is the most common form of thyroid cancer in adults?

Papillary carcinoma

Fibrinogen is....

Part of a CBC in ruminants. Is a part of clotting, Fibrinogen measurements will rise in states of inflammation, makes up about 3-6% of Total Protein measurement

The nurse is assessing a client with multiple myeloma. The nurse should keep in mind that clients with multiple myeloma are at risk for:

Pathologic bone fractures.

Corrected observed retic count formula:

Patient PCV/Divided by Average PCV for species (either dog or cat) X observed retic count (first calculation) = % retics

Patient is to begin taking clopidogrel to prevent thrombus formation what should the healthcare professional question if the patient that has

Peptic ulcer disease

After being in remission from Hodgkin's disease for 18 months, a client develops a fever of unknown origin. The physician orders a blind liver biopsy to rule out advancing Hodgkin's disease and infection. Twenty-four hours after the biopsy, the client has a fever, complains of severe abdominal pain and rigidity, and seems increasingly confused. The nurse suspects that these findings result from:

Perforation of the colon caused by the liver biopsy.

A monocyte is a

Phagocytic leukocyte (WBC) which circulates in the blood for 24 hrs. and then migrates to tissue where it develops into a macrophage.

"A client is in isolation after receiving an internal radioactive implant to treat cancer. Two hours later, the nurse discovers the implant in the bed linens. What should the nurse do first?

Pick up the implant with long-handled forceps and place it in a lead-lined container.

After a transsphenoidal adenohypophysectomy, a client is likely to undergo hormone replacement therapy. A transsphenoidal adenohypophysectomy is performed to treat which type of cancer?

Pituitary carcinoma

If you spun down a BTT what would the fluid portion be?

Plasma

Platelets are important for?

Platelets, or thrombocytes are small, irregularly shaped clear cell fragments (i.e. cells that do not have a nucleus), which are derived from fragmentation of precursor megakaryocytes. The average lifespan of a platelet is normally just 5 to 9 days. Platelets are a natural source of growth factors. They circulate in the blood of mammals and are involved in hemostasis, leading to the formation of blood clots. If the number of platelets is too low, excessive bleeding can occur. However, if the number of platelets is too high, blood clots can form (thrombosis), which may obstruct blood vessels

RBC Descriptions: RBC is bluish in color

Polychromatic

The nurse is interviewing a client about his past medical history. Which preexisting condition may lead the nurse to suspect that a client has colorectal cancer?

Polyps

Which nursing intervention is most appropriate for a client with multiple myeloma

Preventing bone injury

PLE is an abbreviation for....

Protein Losing Enteropathy...GI system releasing proteins

PLN is an abbreviation for....

Protein Losing Nephropathy....kidneys releasing proteins into the urine

A client with a cancerous tumor must undergo a modified radical mastectomy, which includes axillary node removal and immediate breast reconstruction. The nurse explains to the client that the axillary nodes will be removed in order to:

Provide prognostic information.

After a physician explains the risks and benefits of a clinical trial to a client, the client agrees to participate. Later that day, the client requests clarification of the process involved in the clinical trial. As a member of the multidisciplinary team, how should the nurse respond?

Provide the information requested.

During chemotherapy, an oncology client has a nursing diagnosis of Impaired oral mucous membrane related to decreased nutrition and immunosuppression secondary to the cytotoxic effects of chemotherapy. Which nursing intervention is most likely to decrease the pain of stomatitis?

Providing a solution of hydrogen peroxide and water for use as a mouth rinse

What is evaluated in the Body of a blood film @ 10X?

RBC associations: Rouleaux & Agglutination

Biochemistries

RTT which contains no anticoagulant

A client with vaginal cancer asks the nurse, "What is the usual treatment for this type of cancer?" Which treatment should the nurse name?

Radiation therapy

What does it mean if a cell is Normocytic?

Red blood cells that are normal in size and usually also in hemoglobin content

A client has been receiving chemotherapy to treat cancer. Which data collection finding suggests that the client has developed stomatitis (inflammation of the mouth)?

Red, open sores on the oral mucosa

"The nurse is caring for a client who has just had a modified radical mastectomy with immediate reconstruction. She's in her 30s and has two young children. Although she's worried about her future, she seems to be adjusting well to her diagnosis. What should the nurse do to support her coping?

Refer the client to the American Cancer Society's Reach for Recovery program or another support program.

Plasma proteins are measured by what device? How accurate is this device?

Refractometer - accurate to 0.2g/dl

A client with a cerebellar brain tumor is admitted to an acute care facility. The nurse assists with formulating a nursing diagnosis of Risk for injury. Which "related-to" phrase is appropriate for the nurse to add to complete the nursing diagnosis statement?

Related to impaired balance

FALSE

Relative polycythemia is most often caused by hypoxia without anemia

What is the most important postoperative instruction the nurse must give a client who has just returned from the operating room after receiving a subarachnoid block?

Remain supine for the time specified by the physician.

A nurse is planning care for a client who has cancer and has developed thrombocytopenia following chemotherapy. Which of the following precautions should the nurse offer to minimize the adverse effects of thrombocytopenia? Monitor visitors for manifestations of infection. Remind the client to use an electric razor. Encourage frequent rest periods. Instruct the client to rinse mouth daily with normal saline.

Remind the client to use an electric razor. · Thrombocytopenia is a decrease in the client's blood platelet count, which places the client at an increased risk of bleeding due to the blood's inability to clot. Therefore, the nurse should institute bleeding precautions, which includes the use of an electric razor.

A client with uterine cancer is receiving chemo and develops thrombocytopenia. Due to this side effect, the nurse should plan to give the greatest consideration to

Reminding the client to use an electric razor

A client with uterine cancer is receiving chemo and develops thrombocytopenia. Due to this side effect, the nurse should plan to give the greatest consideration to-- a. monitoring visitors for signs of infection b. reminding the client to use an electric razor c. encouraging frequent rest periods d. instruct client to vigorously brush and floss teeth

Reminding the client to use an electric razor. Thrombocytopenia is a decrease in the client's platelet count, which places the client at an increased risk of bleeding due to an inability to clot.

FALSE

Renal failure will cause a relative decrease in PCV

A client in the terminal stage of cancer is receiving a continuous infusion of morphine (Duramorph) for pain management. Which data collection finding suggests that the client is experiencing an adverse effect of this drug?

Respiratory rate of 8 breaths/minute

Patient is to begin taking folic acid to treat megaloblastic anemia what is the laboratory value needed

Reticulocyte count

A nurse is developing a care plan for a client immediately following a bone marrow transplant. What post transplant concerns should be the nurse's highest priority?

Risk for infection

After being seen in the oncology clinic, a client is admitted to the hospital with severe bone marrow depression. The client's cancer therapy consisted of radiation and chemotherapy. Which nursing diagnosis takes priority when developing this client's plan of care?

Risk for infection

A nurse is developing a care plan for a client immediately following a bone marrow transplant. What post transplant concerns should be the nurse's highest priority? a. pain b. social isolation c. risk for infection d. graft-versus- host disease

Risk for infection. the major cause of death in the first week to 10 days following a bone marrow transplant is infection. Priority concern should be maintaining a micro organism free environment.

List the characteristics/distribution with each of the WBC's Lymphocyte:

Round to oval nucleus, High nucleus to cytoplasm ratio, agranulocyte, most abundant cell in a cow, 55-85% of the differential in a cow

What use is there for the Black Ring Crit Tube? What anticoagulant does the black ring crit tube contain?

Same use as the Red Ring Crit Tube only this one is smaller in diameter and is used for blood samples from avians and reptiles for a CBC This also contains the anticoagulant Heparin

RBC Descriptions: RBC fragments

Schizocytes

List the characteristics/distribution with each of the WBC's Neutrophil:

Segmented nucleus with dense chromatin, clear to dusty purple grey granules, granulocyte, most abundant cell in dogs & cats, 55-85% of the differential in dogs & cats.

If you spun down a RTT what would the fluid portion be?

Serum

Creatinine is....

Serum creatinine (a blood measurement) is an important indicator of renal health because it is an easily-measured by-product of muscle metabolism. Creatinine itself is an important biomolecule because it is a major by-product of energy usage in muscle. Creatine is primarily synthesized in the liver then transported through blood to the other organs, muscle, and brain. Creatinine is chiefly filtered out of the blood by the kidneys. The test is done to see how well your kidneys work. Creatinine is removed from the body entirely by the kidneys. If kidney function is abnormal, creatinine levels will increase in the blood (because less creatinine is released through your urine).

"A client with stage II ovarian cancer undergoes a total abdominal hysterectomy and bilateral salpingo-oophorectomy with tumor resection, omentectomy, appendectomy, and lymphadenectomy. During the second postoperative day, which data collection finding would raise concern in the nurse?

Shallow breathing and increasing lethargy

A 35-year-old female client is requesting information about mammograms and breast cancer. She isn't considered at high risk for breast cancer. What should the nurse tell this client?

She should eat a low-fat diet to further decrease her risk of breast cancer.

On discharge, a client who underwent a left mastectomy expresses relief that "the cancer" has been treated. When discussing this issue with the client, the nurse should stress that she:

Should continue to perform breast self-examination on her right breast

4 Criteria by which we assess RBC Morphology:

Size, Shape, Color, +/- Inclusions

RBC Descriptions: May occur in dogs with anemia, more contents than membrane

Spherocyte

What is the conventional time for a PCV?

Spin for 5 minutes @ 6000 RPM's

Explain Red Blood Cell development/production:

Stimulus for RBC production is HYPOXIA, kidneys produce EPO in response to the Hypoxia which tells the Pluiripotential Stem Cells in the bone marrow to begin the maturing process to become red blood cells (RBC's)

RBC Descriptions: RBC that may occur in a dwarf

Stomatocyte

A nurse is monitoring a client who has cancer and is receiving chemotherapy by peripheral IV infusion. The client reports pain at the insertion site and the nurse notes fluid leaking around the catheter. Which of the following actions should the nurse take first? Take a photograph of the peripheral IV site. Obtain and record the client's vital signs. Stop the infusion. Identify all medications administered through the IV site for the past 24 hr.

Stop the infusion. · The nurse should apply the urgent versus nonurgent priority-setting framework. Using this framework, the nurse should consider urgent needs the priority need because they pose more of a threat to the client. The nurse might also need to use Maslow's hierarchy of needs, the ABC priority-setting framework, or nursing knowledge to identify which finding is the most urgent. Many chemotherapy medications are vesicants that can cause extensive tissue damage if extravasation occurs; therefore, the nurse's first action should be to stop the infusion immediately.

A nurse is preparing a client for discharge after a prolonged hospitalization in which the client had a colon resection and colostomy formation for treatment of colon cancer. The client's family has concerns about managing his care at home. Which factor is most important in ensuring successful home care?

Support from friends and family

Acathocytes and targets can be associated with liver problems

TRUE

An animal with an iron deficiency will have a microcytic hypochromic non-regenerative anemia

TRUE

An animal with hemolysis will have an artifactual hyperproteinemia

TRUE

Geriatrics will have a regenerative hyperproteinemia

TRUE

If an animal has an increase in retics, you would expect to see polychromasia and anisocytosis in routine stain

TRUE

Lymphocytes stimulus for production is antigens and they produce antibodies

TRUE

MPS is macrophages in the liver, spleen, and the bone marrow

TRUE

Overhydration will cause a relative hypoproteinemia

TRUE

Parasites with long tissue migration will cause an increase in eosinophils

TRUE

Polycythemia vera is a RBC tumor and there is no increase in EPO

TRUE

Secondary absolute polycythemia is caused by glucocorticoids or hypoxia without anemia

TRUE

The 3 mechanisms of anemia are decreased production (kidney failure), increased loss (hemorrhage) & increased destruction (IMHA)

TRUE

There is NO evaluation done in the body or the feathered edge @ 40X & 100X true or false?

TRUE

True or false....A retic (reticulocyte) is a red blood cell?

TRUE - remember that a reticulocyte is a polychromatophilic RBC simply renamed due to it being stained with New Methyline Blue (NMB)

Absolute is defined as...

TRUE or actual deviation/change in the amount of whatever we are measuring

A nurse is preping to discharge a client who had a right radical mastectomy 2 days ago and will be going home with drains still in the incision. When developing a discharge plan of care for this client, what should the nurse include?- a. empty the reservoir bulb attached to the drain once a day b. void exercises that raise the right arm over the head for 6 weeks c. take your BP on the left arm d. report numbness of the inner right arm to the provider

Take your blood press one on the left arm

A nurse is preparing to discharge a client who had a right radical mastectomy 2 days ago and will be going home with drains still in the incision. When assisting in the development of a discharge plan of care for this client, which of the following is appropriate to include?

Take your blood pressure on the left arm

An adolescent female client who had a successful bone marrow transplant is being discharged. What information should the nurse include as a part of the discharge plan? (select all that apply)

Take your temp BID It is important to always wear shoes Avoid tampons

An adolescent female client who had a successful bone marrow transplant is being discharged. What info should the nurse include as a part of the discharge plan? (select all that apply) a. take your temp BID b. you may return to school if you feel strong enough c. it is important to always wear shoes d. clean your toothbrush weekly with isopropyl alcohol e. avoid tampons

Take your temperature twice a day. a temperature great than 101 should be reported. important to always wear shows. to prevent injury and infection. avoid tampons. can support the growth of bacteria if left in too long.

A nurse is caring for a client following a right modified radical mastectomy. Which of the following actions by an assistive personnel (AP) should indicate to the nurse that further teaching is needed?

Taking blood pressure on the right arm

RBC Descriptions: A special type of leptocyte

Target Cell

A client was recently enrolled in a clinical trial for lung cancer. The client's health insurance provider asks the nurse caring for the client about the client's status, treatment regimen, and possible adverse effects of the medication she is taking. How can the nurse best respond?

Tell the provider that a Certificate of Confidentiality was issued; therefore, no information can be released.

The nurse is teaching a male client to perform monthly testicular self-examinations. Which of the following points would be appropriate to make?

Testicular cancer is a highly curable type of cancer.

These three organs are collectively known as what?

The MPS (Mononuclear Phagocytic System)

Why do we have an absolute retic test (count)?

The absolute retic count is a really useful parameter for determining or deciding about auto-immune haemolytic anaemia

A 65-year-old client is scheduled for a right lower lobectomy for lung cancer. During the admission assessment, the client asks for information about a living will and advance directive. The nurse knows that the client understands her teaching about the living will when he says:

The advance directive allows me to state my health care wishes while I'm still able to do so.

Alk Phosphate is a test for?

The alkaline phosphatase test (ALP) is used to help detect liver disease or bone disorders. In conditions affecting the liver, damaged liver cells release increased amounts of ALP into the blood. This test is often used to detect blocked bile ducts because ALP is especially high in the edges of cells that join to form bile ducts. If one or more of them are obstructed, for example by a tumor, then blood levels of ALP will often be high.

A GRAY TOP TUBE is used for what? What does the GTT (Gray Top Tube) contain?

Used for blood glucose level determinations Contains the anti-coagulant OXYLATE (NH4 or K) which binds with the calcium to prevent coagulation

A Blue Ring Hematocrit Tube is used for what? What anticoagulant does the blue ring crit tube contain?

Used in conjunction with the LTT (Lavendar Top Tube) for conducting PCV (packed cell volume), TP (total protein) and blood film, cannot be used by itself There is no anticoagulant agent in a blue ring crit tube

A nurse is obtaining a health history from a client who has cancer of the cervix. Which of the following manifestations should the nurse expect? Weight gain Oliguria Vaginal bleeding Back pain

Vaginal bleeding

A client with advanced breast cancer is prescribed tamoxifen (Nolvadex). When teaching the client about this drug, the nurse should emphasize the importance of reporting which adverse reaction immediately?

Vision changes

A healthcare professional should assess the patient was megaloblastic anemia for indications of which of the following vitamin deficiency

Vitamin B 12 Pt who have megaloblastic anemia have a deficiency of vit. B12, folic acid or both. Cyanocobalamin (Nascobal) treats moderate vit B12 deficiencies. Pt. who have a severe vit. B12 deficiency should take cyanocobalamin and folic acid.

Patient is taking warfarin and arrives at the emergency department and reports rectal bleeding what of the following drugs should be available

Vitamin K

What is one looking for in the monolayer of a blood film @ 40X?

WBC Differential & Begin Morphology

FALSE

WBC increase in ruminates prior to fibrinogen in inflammation

What is one looking for in the monolayer of a blood film @ 100x?

WBC morphology, platelet estimate, RBC Morphology, Platelet Morphology

In horses in states of inflammation

WBC's & Fibrinogen raise equally

In dogs & cats in states of inflammation

WBC's increase first then fibrinogen levels increase

A client received chemotherapy 24 hours ago. Which precautions are necessary when caring for this client?

Wearing personal protective equipment when handling blood, body fluids, or feces

Before a cancer client receiving total parenteral nutrition (TPN) resumes a normal diet, the nurse teaches him about dietary sources of minerals. Which foods are good sources of zinc?

Whole grains and meats

A healthcare professional is caring for a patient who is about to begin taking alteplase to treat acute mycardial infarction. the healthcare professional should understand the drug is most effective when the patient receives it

With in three hours of symptom onset

A client in the final stages of terminal cancer tells his nurse: ""I wish I could just be allowed to die. I'm tired of fighting this illness. I have lived a good life. I only continue my chemotherapy and radiation treatments because my family wants me to."" What is the nurse's best response?

Would you like to meet with your family and your physician about this matter?

The nurse is providing breast cancer education at a community facility. The American Cancer Society recommends that women get mammograms:

Yearly after age 40.

What techniques are used to evaluate TP (total protein)?

You are not able to distinguish/seperate out specific proteins. 1. Refractometer 2. Colorimetric (automated machine) 3. Fibrinogen measurement which is a precursor to fibrin (for clotting)

"A 43-year-old black client without a family history of prostate cancer asks the nurse when he should have a prostate-specific antigen (PSA) test and a digital rectal examination (DRE) performed. Which response by the nurse is appropriate?

You should start undergoing prostate cancer screening at age 45.

A "Mechanism" for something is defined as..

a direct cause of/for the change

Polychromasia is... If you have Polychromasia in your RBC morphology you must also have...

a disorder where there is an abnormally high number of red blood cells found in the bloodstream as a result of being prematurely released from the bone marrow during blood formation. These cells are often shades of grayish blue. Polychromasia is usually a sign of bone marrow stress as well as immature red blood cells Anisocytosis

A client with multiple myeloma is admitted to the unit with WBC of 2200. What foods should the nurse prohibit family members from bringing to the client? a. fried chicken from a fast food shop b. a gift basket of bagels c. factory-sealed box of chocolates d. a fresh fruit basket

a fresh fruit basket. raw fruits and vegetables are contraindicated for this client since the skin may harbor bacteria that can cause an infection in this client. these foods should not be brought into the clients room or consumed by the client.

Thrombocytopenia

a low number of platelets

A nurse is caring for a client who has an abdominal aortic aneurysm and is scheduled for surgery. The clients vital signs are BP 160/98, HR 102/min, R 22/min, SpO2 95%. Which of the following actions should the nurse take? a) Administer antihypertensive medication for BP b) Monitor that urinary output is 20 ml/hr c) Withhold pain meds to prepare for surgery d) Take vital signs every 2 hours

a) Administer antihypertensive medication for the elevated BP because HTN can cause a sudden rupture of the aneurysm due to pressure on the arterial wall.

A nurse is assessing a client who has fluid volume overload from a cardiovascular disorder. Which of the following manifestations should the nurse expect? (Select All That Apply) a) Jugular vein distension b) Moist crackles c) Postural hypotension d) Increased heart rate e) Fever

a) Jugular vein distention : The increase in venous pressure due to excessive circulating blood volume results in neck vein distention b) Moist crackles : An indicator of pulmonary edema that can quickly lead to death d) Increased heart rate : fluid volume excess, or hypervolemia, is an expansion of fluid volume in the extracellular fluid compartment. This results in increased heart rate and bounding pulses

A nurse is completing a medication history for a client who reports using fish oil as a dietary supplement. Which of the following substances in fish oil should the nurse recognize as a health benefit to the client? a) Omega-3 fatty acids b) Antioxidants c) Vitamins A, D and C d) Beta-carotene

a) Omega-3 fatty acids Fish oil contains omega-3 fatty acids, which can help lower the risk of cardiovascular disease and stroke by decreasing triglyceride levels

A nurse is reviewing a client's repeat lab results 4 hrs after administering fresh frozen plasma (FFP). Which of the following lab results should the nurse review? a) Prothrombin time b) WBC count c) Platelet count d) Hematocrit

a) Prothrombin time The nurse should review the clients prothrombin time after the administration of FFP, which is plasma rich in clotting factors. FFP is administered to treat acute clotting disorders. The desired effect is a decrease in the prothrombin time

A nurse is caring for a client who has a demand pacemaker inserted with the rate set at 72/min. Which of the following findings should the nurse expect? a) Telemetry monitoring shows QRS complexes occurring at a rate of 74/min with no pacing spikes b) The client is experiencing premature ventricular complexes at 12/min c) Telemetry monitoring shows pacing spikes with no QRS complexes d) The client is experiencing hiccups

a) Telemetry monitoring shows QRS complexes occurring at a rate of 74/min with no pacing spikes. The nurse should not expect pacer spikes when the client's pulse is greater than the set rate of 72/min, because the client's intrinsic rate overrides the set rate of the pacemaker.

A nurse is assessing a client who has late-stage heart failure and is experiencing fluid volume overload. Which of the following findings should the nurse expect? a) weight gain 1 kg (2.2 lb) in 1 day b) Pitting edema +1 c) Client report of nocturnal cough d) B-Type Natriuretic Peptide (BNP) level of 100 pg/mL

a) weight gain 1kg (2.2lbs) in 1 day A weight gain of 2.2 lbs in 1 day alerts the nurse that the client is retaining fluid and is at risk of fluid volume overload. This is an indication that the client's heart failure is worsening.

Of the following, the disease most closely associated with cytoplasmic granule fusion is: a. Chédiak-Higashi syndrome b. Pelger-Huet anomaly c. May-Hegglin anomaly d. Alder-Reilly anomaly

a. Chédiak-Higashi syndrome

A nurse is assessing a client who is postoperative and has anemia due to excess blood loss following surgery. Which of the following findings should the nurse expect? a. Fatigue b. Hypertension c. Bradycardia d. Diarrhea

a. Fatigue Fatigue The nurse should identify that the client who has anemia due to blood loss following surgery will experience fatigue. This is due to the body's decreased ability carry oxygen to vital tissues and organs. Constipation is a manifestation of anemia due to blood loss following surgery. Tachycardia is manifestation of anemia due to blood loss following surgery. Hypotension is manifestation of anemia due to blood loss following surgery.

Of the following, the disease most closely associated with glucocerebrosidase deficiency is: a. Gaucher's disease b. Chédiak-Higashi syndrome c. Pelger-Huet anomaly d. May-Hegglin anomaly

a. Gaucher's disease

A 40-year-old man had an erythrocyte count of 2.5 x l06/uL, hematocrit of 22%, and a reticulocyte count of 2.0%. Which of the following statements best describes his condition? a. The absolute reticulocyte count is 50 x 103/uL, indicating that the bone marrow is not adequately compensating for the anemia. b. The reticulocyte count is greatly increased, indicating an adequate bone marrow response for this anemia. c. The absolute reticulocyte count is 500 x l03/uL, indicating that the bone marrow is adequately compensating for the anemia. d. The reticulocyte count is slightly increased, indicating an adequate response to the slight anemia.

a. The absolute reticulocyte count is 50 x 103/uL, indicating that the bone marrow is not adequately compensating for the anemia.

Plasma from a patient with lupus coagulation inhibitor can show: a. a prolonged APTT and normal PT b. may exhibit bleeding tendencies c. no change with platelet neutralization d. complete correction when incubated with normal plasma

a. a prolonged APTT and normal PT

Prothrombin is: a. a protein formed by the liver in the presence of vitamin K b. an enzyme that converts fibrinogen into fibrin threads c. the end product of the reaction between fibrinogen and thrombin d. a protein released by platelets during coagulation

a. a protein formed by the liver in the presence of vitamin K

Platelet satellitosis is usually due to: a. abnormal proteins b. inadequate mixing of blood and anticoagulant c. hemorrhage d. poorly made wedge smear

a. abnormal proteins

A 54-year-old man was admitted with pulmonary embolism and given streptokinase. Which of the following would be most useful in monitoring this therapy? a. activated partial thromboplastin time b. bleeding time c. prothrombin time d. thrombin time

a. activated partial thromboplastin time

Most childhood leukemias are: a. acute lymphocytic b. acute monocytic c. chronic myelocytic d. chronic lymphocytic

a. acute lymphocytic

Specific (secondary) granules of the neutrophilic granulocyte: a. appear first at the myelocyte stage b. contain lysosomal enzymes c. are formed on the mitochondria d. are derived from azurophil (primary) granules

a. appear first at the myelocyte stage

Which of the following measures platelet function? a. bleeding time b. prothrombin time c. thrombin time d. partial thromboplastin time

a. bleeding time

Which of the following platelet responses is most likely associated with classic von Willebrand's disease? a. decreased platelet aggregation to ristocetin b. normal platelet aggregation to ristocetin c. absent aggregation to epinephrine, ADP, and collagen d. decreased amount of ADP in platelets

a. decreased platelet aggregation to ristocetin

In polycythemia vera, the hemoglobin, hematocrit, red blood cell count, and red cell mass are: a. elevated b. normal c. decreased

a. elevated

A patient with thalassemia minor characteristically has a(n): a. elevated A2 hemoglobin b. low fetal hemoglobin c. high serum iron d. normal red cell fragility

a. elevated A2 hemoglobin

What completes the circuit between the stationary and the moving electrodes in the fibrometer? a. fibrin strand b. cam sensing device c. third electrode d. turbidity of reaction

a. fibrin strand

If a clot incubated at 37°C dissolves within 24 hours, which of the following should be suspected? a. fibrinolysins b. low fibrinogen level c. Factor VIII deficiency d. thrombocytopenia

a. fibrinolysins

Which of the following is characteristic of Bernard-Soulier syndrome? a. giant platelets b. normal bleeding time c. abnormal aggregation with ADP d. increased platelet count

a. giant platelets

Acute disseminated intravascular coagulation is characterized by: a. hypofibrinogenemia b. thrombocytosis c. negative D-dimer d. shortened thrombin time

a. hypofibrinogenemia

When using an electronic cell counter, which of the following results can occur in the presence of a cold agglutinin? a. increased MCV and decreased RBC b. increased MCV and normal RBC c. decreased MCV and increased MCHC d. decreased MCV and RBC

a. increased MCV and decreased RBC

Biochemical abnormalities characteristic of polycythemia vera include: a. increased serum B12 binding capacity b. hypouricemia c. hypohistaminemia d. decreased leukocyte alkaline phosphatase activity

a. increased serum B12 binding capacity

The Philadelphia chromosome is formed by a translocation between the: a. long arm of chromosome 22 and long arm of chromosome 9 b. long arm of chromosome 21 and long arm of chromosome 9 c. long arm of chromosome 21 and long arm of chromosome 6 d. long arm of chromosome 22 and long arm of chromosome 6

a. long arm of chromosome 22 and long arm of chromosome 9

Which of the following cells is the largest cell in the bone marrow: a. megakayocyte b. histiocyte c. osteoblast d. mast cell

a. megakayocyte

Which of the following is associated with Chédiak-Higashi syndrome? a. membrane defect of lysosomes b. Döhle bodies and giant platelets c. two-lobed neutrophils d. mucopolysaccharidosis

a. membrane defect of lysosomes

Which of the following is a characteristic of Factor XII deficiency? a. negative bleeding history b. normal clotting times c. decreased risk of thrombosis d. epistaxis

a. negative bleeding history

Which of the following are found in association with megaloblastic anemia? a. neutropenia and thrombocytopenia b. decreased LD activity c. increased erythrocyte folate levels d. decreased plasma bilirubin levels

a. neutropenia and thrombocytopenia

The most appropriate screening test for hereditary spherocytosis is: a. osmotic fragility b. sucrose hemolysis c. heat instability test d. Kleihauer-Betke

a. osmotic fragility

Peripheral blood smears from patients with untreated pernicious anemia are characterized by: a. pancytopenia and macrocytosis b. pancytopenia and leukocytosis c. leukocytosis and ovalocytosis d. pancytopenia and microcytosis

a. pancytopenia and macrocytosis

Decreased to normal erythropoietin production is most likely to be associated with: a. polycythemia vera b. polycythemia, secondary to hypoxia c. relative polycythemia associated with dehydration d. polycythemia associated with renal disease

a. polycythemia vera

A nurse is caring for a client who receives furosemide (Lasix) to treat heart failure. Which of the following laboratory values should the nurse monitor for this client due to this medication? a. potassium b. albumin c. cortisol d. bicarbonate

a. potassium Furosemide is a loop diuretic that promotes the excretion of potassium. The nurse should monitor the client's potassium level to watch for hypokalemia. Furosemide does not affect albumin levels. Furosemide does not affect cortisol levels, although it can lower serum sodium levels. Furosemide does not affect bicarbonate levels.

Aspirin affects platelet function by interfering with platelets' metabolism of: a. prostaglandins b. lipids c. carbohydrates d. nucleic acids

a. prostaglandins

Elevation of the lymphocyte percentage above 47% is termed: a. relative lymphocytosis b. absolute lymphocytosis c. leukocytosis d. absolute neutrophilic leukocytosis

a. relative lymphocytosis

Supravital staining is important for reticulocytes since the cells must be living in order to stain the: a. remaining RNA in the cell b. iron before it precipitates c. cell membrane before it dries out d. denatured hemoglobin in the cell

a. remaining RNA in the cell

A phase-platelet count was performed and the total platelet count was 356,000/uL. Ten fields on the stained blood smear were examined for platelets and the results per field were: 16, 18, 15, 20, 19, 17, 19, 18, 20, 16 The next step would be to: a. report the phase-platelet count since it correlated well with the slide b. repeat the phase-platelet count on a re-collected specimen and check for clumping c. check 10 additional fields on the blood smear d. repeat the platelet count using a different method

a. report the phase-platelet count since it correlated well with the slide

Higher levels of employee motivation occur when supervisor: a. sets goals to be accomplished b. provides all the details of the task c. constantly monitors progress d. immediately corrects every error

a. sets goals to be accomplished

A red blood cell about 5 um in diameter that stains bright red and shows no central pallor is a: a. spherocyte b. leptocyte c. microcyte d. macrocyte

a. spherocyte

List the characteristics/distribution with Monocyte:

ameboid nucleus w/open "lacy" chromatin, equal or low nucleus to cytoplasm ratio, 0-5% in all species.

Artifact is defined as...

amount is the same but something is mimicking the item EX: dirty refractometer

Hypoproteinemia can be caused by an absolute proteinemia which is...

an actual decrease in protein due to any of these 3 mechanisms (causes) 1. lower production due to liver failure or immune deficiency 2. increased loss due to hemorhage, external loss, starvation 3. decreased consumption due to malabsorption, maldigestion, irritable bowel syndrome

Absolute proteinemia can be caused by..

an increase in production of protein due to early gestation, inflammation, immune mediated diseases (Lupus) or protein producing neoplasias

Polycythemia is the extreme opposite of anemia. It's defined as...

an increase in the hematocrit, hemoglobin concentration & RBC counts. Polycythemia refers to any increase (overproduction) in red blood cells and is a disease state in which the proportion of blood volume that is occupied by red blood cells increases. FISH IN LAKE EXAMPLE

Thrombocytosis

an increase in the number of platelets

Relative proteinemia can be caused by

an increased concentration of protein due to dehydration

Hemoglobin (Hb) is...

an iron rich protein that gives blood its red color. This protein helps RBC's carry O2 from the lungs to the rest of the body

Dysproteinemias is defined as....

any abnormality of the protein content of the blood out of the "normal" range either high or low

A client with bladder cancer is being treated with intravesical administration of immunotherapy. During treatment, the nurse should- a. maintain the client in the Trendelenburg position for at least 2 hr b. tell client to get up to urinate when he feels the urge c. assist the client with changing positions ever 15 mins for 2 hr d. have a gown and mask ready for removal of the fluid from the bladder

assist the client with changing positions every 15 minutes for 2 hours.

A nurse is caring for a client who has hemophilia. The client reports pain and swelling in a joint following an injury. Which of the following actions should the nurse take? a) Obtain blood samples to test platelet function b) Prepare for replacement of the missing clotting factor c) Administer aspirin for the client's pain d) Place the bleeding joint in the dependent position

b) Prepare for replacement of the missing clotting factor Hemophilia is a hereditary bleeding disorder in which blood clots slowly and abnormal bleeding occurs. It is caused by a deficiency in the most common clotting factor, factor VIII (hemophilia A). Aggressive factor replacement is initiated to prevent hemarthrosis that can result in long-term loss of range of motion in repeatedly affected joints.

A nurse is transfusing a unit of O-negative fresh frozen plasma to a client whose blood type is B positive. Which of the following actions should the nurse take? a) Continue to monitor for manifestations of a transfusion reaction b) Remove the unit of plasma immediately and start an IV infusion of normal saline solution c) Continue the transfusion and repeat the type and crossmatch d) Prepare to administer a dose of diphenhydramine IV

b) Remove the unit of plasma immediately and start an IV infusion of normal saline solution A client who receives FFB that is not compatible can experience a hemolytic transfusion reaction. The nurse should stop the transfusion and infuse 0.9% sodium chloride solution with new tubing.

A nurse is administering a unit of packed RBCs to a client who is postoperative. The client reports itching and has hives 30 min after the infusion begins. Which of the following actions should the nurse take first? a) Maintain the IV access with 0.9% sodium chloride b) Stop the infusion of blood c) send the blood container and tubing to the blood bank d) Obtain a urine sample

b) stop the infusion of blood The nurse should apply the urgent vs nonurgent priority setting framework. The nurse should stop the infusion of blood because the client has manifestations of an allergic reaction.

What is the MCHC if the hematocrit is 20%, the RBC is 2.4 x 106/uL, and the hemoglobin is 5 g/dL? a. 21% b. 25% c. 30% d. 34%

b. 25%

The atypical lymphocyte seen in the peripheral smear of patients with infectious mononucleosis is reacting to which of the following? a. T lymphocytes b. B lymphocytes c. monocytes d. mast cells

b. B lymphocytes

A nurse is caring for a client who has infective endocarditis. Which of the following manifestation is the priority for the nurse to monitor for? a. Anorexia b. Dyspnea c. Fever d. Malaise

b. Dyspnea When using the airway, breathing, circulation (ABC) approach to client care, the nurse determines the priority manifestation to monitor for is dyspnea. Dyspnea can be an indication of left-sided heart failure, or a pulmonary infarction due to embolization. Anorexia is a manifestation associated with infective endocarditis; however, another manifestation is a greater risk to the client, and therefore the priority. Fever is a manifestation associated with infective endocarditis; however, another manifestation is a greater risk to the client, and therefore the priority. Malaise is a manifestation associated with infective endocarditis; however, another manifestation is a greater risk to the client, and therefore the priority.

Which of the following is associated with May-Hegglin anomaly? a. membrane defect of lysosomes b. Döhle bodies and giant platelets c. chronic myelogenous leukemia d. mucopolysaccharidosis

b. Döhle bodies and giant platelets

A nurse in a cardiac unit is caring for a client with acute right-sided heart failure. which of the following findings should the nurse expect? a. decreased BNP b. elevated CVP c. increased PAWP d. decreased specific gravity

b. Elevated CVP CVP is a measurement of the pressure in the right atria or ventricle at the end of diastole. An elevated CVP is indicative of heart failure. The BNP is a neurohormone that aids in the regulation of fluid balance by detecting increased stretch of the myocardium and triggering diuresis through sodium excretion via the kidneys. The BNP level is elevated in the client who has acute heart failure. Pulmonary pressure increases in left-sided heart failure because of the increased pressure and volume of blood in the left ventricle. Urinary specific gravity is increased in the client who has heart failure as a result of fluid retention by the body.

A patient has a history of mild hemorrhagic episodes. Laboratory results include a prolonged prothrombin time and activated partial thromboplastin time. The abnormal prothrombin time was corrected by normal and adsorbed plasma, but not aged serum. Which of the following coagulation factors is deficient? a. prothrombin b. Factor V c. Factor X d. Factor VII

b. Factor V

Which of the following factor deficiencies is associated with either no bleeding or only a minor bleeding tendency, even after trauma or surgery? a. Factor X b. Factor XII c. Factor XIII d. Factor V

b. Factor XII

A nurse is preparing an in-service about the various supplements clients might use. Which of the ff. herbal supplements should the nurse include as potentially increasing the anticoagulant effects of aspirin and other oral anticoagulants? a. Valerian b. Feverfew c. Milk Thistle d. Saw Palmetto

b. Feverfew Feverfew can increase the risk of bleeding due to the suppression of platelet aggregation. Clients can use valerian to promote sleep and decrease restlessness from anxiety. Milk thistle reduces the effectiveness of oral contraceptives. Clients may use saw palmetto to relieve urinary and prostate symptoms, such as frequent urination and nocturia.

the nurse is collecting a medication history from a client who is scheduled to have a cardiac catheterization. Which of the following medications taken by the client interacts with contrast material and places the client at risk for acute kidney injury? a. Atorvastatin b. Metformin c. Nitroglycerin d. Carvedilol

b. Metformin Metformin interacts with contrast dye and can cause acute kidney damage. Atorvastatin is contraindicated for a client who has active hepatic disease, but it does not interact with contrast material. Nitroglycerin is contraindicated for a number of conditions including increased intracranial pressure, but it does not interact with contrast material. Carvedilol is contraindicated for a number of conditions including 2nd and 3rd degree heart block, but it does not interact with contrast material

Of the following, the disease most closely associated with granulocyte hyposegmentation is: Hematology 227 a. May-Hegglin anomaly b. Pelger-Huet anomaly c. Chédiak-Higashi syndrome d. Gaucher's disease

b. Pelger-Huet anomaly

Which of the following is a true statement about acute idiopathic thrombocytopenic purpura (ITP)? a. It is found primarily in adults. b. Spontaneous remission usually occurs within several weeks. c. Women are more commonly affected. d. Peripheral destruction of platelets is decreased.

b. Spontaneous remission usually occurs within several weeks.

Which of the following coagulation factors is considered to be labile? a. II b. V c. VII d. X

b. V

Evidence indicates that the genetic defect in thalassemia usually results in: a. the production of abnormal globin chains b. a quantitative deficiency in RNA resulting in decreased globin chain production c. a structural change in the heme portion of the hemoglobin d. an abnormality in the alpha or beta chain binding or affinity

b. a quantitative deficiency in RNA resulting in decreased globin chain production

A 20-year-old woman with sickle cell anemia whose usual hemoglobin concentration is 8 g/dL develops fever, increased weakness, and malaise. The hemoglobin concentration is 4 g/dL and the reticulocyte count is 0.1%. The most likely explanation for this clinical picture is: a. increased hemolysis due to hypersplenism b. aplastic crisis c. thrombotic crisis d. occult blood loss

b. aplastic crisis

The most frequent type of acute lymphocytic leukemia (ALL) is: a. T-cell childhood b. common childhood c. B-cell childhood d. undifferentiated childhood

b. common childhood

The following results were obtained on an electronic particle counter: -WBC: 6.5x103/uL -RBC: 4.55 x 106/uL -Hgb: 18.0 g/dL -Hct: 41.5% -MCV: 90.1 fL -MCH: 39.6 pg -MCHC: 43.4% The first step in obtaining valid results is to: a. perform a microhematocrit b. correct the hemoglobin for lipemia c. dilute the blood d. replace the lysing agent

b. correct the hemoglobin for lipemia

If a blood smear is dried too slowly, the red blood cells are often; a. clumped b. crenated c. lysed d. destroyed

b. crenated

Which of the following sets of laboratory findings is consistent with hemolytic anemia? a. normal or slightly increased erythrocyte survival; normal osmotic fragility b. decreased erythrocyte survival; increased catabolism of heme c. decreased serum lactate dehydrogenase activity; normal catabolism of heme d. normal concentration of haptoglobin; marked hemoglobinuria

b. decreased erythrocyte survival; increased catabolism of heme

Which of the following will not cause erroneous results when using a phase optical system for enumerating platelets? a. incipient clotting b. decreased hematocrit c. Howell-Jolly bodies d. leukocyte cytoplasmic fragments

b. decreased hematocrit

The hypoproliferative red cell population in the bone marrow of uremic patients is caused by: a. infiltration of bone marrow by toxic waste products b. decreased levels of circulating erythropoietin c. defective globin synthesis d. overcrowding of bone marrow space by increased myeloid precursors

b. decreased levels of circulating erythropoietin

A nurse is assessing a client who has infective endocarditis. Which of the following should be the priority for the nurse to report to the provider? a. Splinter hemorrhages to the nails b. Dyspnea c. fever d. Clusters of petechiae in the mouth

b. dyspnea The client who has infective endocarditis and develops dyspnea, tachycardia, or a cough might be developing heart failure or experiencing pulmonary emboli, two complications of the infection. Fine reddish-brown lines, called splinter hemorrhages, are an expected finding in the client who has infective endocarditis. Clients who have infective endocarditis might experience intermittent fevers, even after initiating antibiotic therapy. Petechiae are manifestations of infective endocarditis and can appear in clusters over different areas of the body, including the mucous membranes of the mouth, the palms of the hands, and soles of the feet.

The M:E ratio in acute granulocytic leukemia is usually: a. normal b. high c. low d. variable

b. high

The M:E ratio in chronic granulocytic leukemia is usually: a. normal b. high c. low d. variable

b. high

A patient has pancytopenia, decreased total serum iron, and decreased serum iron-binding capacity, and shows a homogeneous fluorescence pattern with a high titer on a fluorescent antinuclear antibody test. This is suggestive of: a. polycythemia vera b. lupus erythematosus c. iron deficiency anemia d. hemoglobin SC disease

b. lupus erythematosus

In an uncomplicated case of infectious mononucleosis, which of the following cells are affected? a. erythrocytes b. lymphocytes c. monocytes d. thrombocytes

b. lymphocytes

The cell series most readily identified by a positive Sudan black B is: a. erythrocytic b. myelocytic c. plasmocytic d. lymphocytic

b. myelocytic

The first step in the determination of functional antithrombin III (AT III) is to: a. neutralize plasma antithrombin b. neutralize thrombin with test plasma c. incubate plasma with anti-AT III d. precipitate AT III with plasma

b. neutralize thrombin with test plasma

The granules of Alder-Reilly anomaly will stain positively with: a. Sudan black B b. periodic acid-Schiff c. myeloperoxidase d. naphthol-AS-D chloroacetate esterase

b. periodic acid-Schiff

Which of the following cell types is characteristic of Pelger-Huet anomaly? a. band form b. pince-nez form c. normal neutrophil d. myelocyte

b. pince-nez form

Which of the following types of polycythemia is most often associated with emphysema? a. polycythemia vera b. polycythemia, secondary to hypoxia c. relative polycythemia associated with dehydration d. polycythemia associated with renal disease

b. polycythemia, secondary to hypoxia

The main function of the hexose monophosphate shunt in the erythrocyte is to: a. regulate the level of 2, 3-DPG b. provide reduced glutathione to prevent oxidation of hemoglobin c. prevent the reduction of heme iron d. provide energy for membrane maintenance

b. provide reduced glutathione to prevent oxidation of hemoglobin

Which of the following characteristics are common to hereditary spherocytosis, hereditary elliptocytosis, hereditary stomatocytosis, and paroxysmal nocturnal hemoglobinuria? a. autosomal dominant inheritance b. red cell membrane defects c. positive direct antiglobulin test d. measured platelet count

b. red cell membrane defects

A citrated blood specimen for coagulation studies is to be collected from a polycythemic patient. The anticoagulant should be: a. the standard volume b. reduced in volume c. changed to EDTA d. changed to oxalate

b. reduced in volume

A phase-platelet count is performed using a red cell pipet. 155 platelets are counted on one side of the hemacytometer in the red cell counting area, and 145 are counted on the other side in the same area. After making the appropriate calculations, the next step would be to: a. repeat the procedure, using a 1:20 dilution in a white cell pipet b. report the calculated value c. collect a new specimen d. repeat the procedure, using a 1:200 dilution in the red cell pipet

b. report the calculated value

Polychromatic red cells when stained with a supravital stain are called: a. siderocytes b. reticulocytes c. schistocytes d. spherocytes

b. reticulocytes

An automated platelet count is 15 x 103/uL. The technologist should: a. phone result to physician immediately b. review peripheral smear c. report count d. repeat collection

b. review peripheral smear

The characteristic morphologic feature in multiple myeloma is: a. cytotoxic T cells b. rouleaux formation c. spherocytosis d. macrocytosis

b. rouleaux formation

The most appropriate screening test for paroxysmal nocturnal hemoglobinuria is: a. heat instability test b. sucrose hemolysis c. osmotic fragility d. dithionite solubility

b. sucrose hemolysis

Concentration of proteins in the blood is a circular loop...Explain

balance between uptake of proteins in the GI tract into circulatory system, filtration through capillaries into tissues (cells) and a return of proteins from the tissues into lymphatic system and back into circulatory system. This is why Total Protein "normal" ranges are small. "Normals" assume an adult animal with no physiological changes

Why does a baby animal (newborn) have hypoproteinemia?

because they have not been exposed to immunoglobulins (colostrum) long enough yet to raise the protein levels. Will take 6-12 months till they fall into "normal" range.

Causcasian women over the age of 60 are at an increased risk for what type of cancer?

breast

A nurse is assessing for cardiac tamponade on a client who had coronary artery bypass grafts. Which of the following actions should the nurse take? a) Check for hypertension b) Auscultate for loud, bounding heart sounds c) Auscultate blood pressure for pulsus paradoxus d) Check for a pulse deficit

c) Auscultate blood pressure for pulsus paradoxus The client who has cardiac tamponade will have pulsus paradoxus when the systolic blood pressure is at least 10 mm Hg higher on expiration that on inspiration. This occurs because of the sudden decrease in cardiac output from the fluid compressing the atria and ventricles

A nurse in a clinic is assessing the lower extremities and ankles of a client who has a history of peripheral arterial disease. Which of the following findings should the nurse expect? a) Pitting edema b) Areas of reddish-brown pigmentation c) Dry, pale skin with minimal body hair d) Sunburned appearance with desquamation

c) Dry, pale skin with minimal body hair A client who has peripheral arterial disease can display dry, scaly, pale, or mottled skin with minimal body hair because of narrowing of the arteries in the legs and feet. This causes a decrease in blood flow to the distal extremities, which can lead to tissue damage. Common manifestations are intermittent claudication (leg pain with exercise), cold or numb feet at rest, loss of hair on lower legs, and weakened pulses.

A nurse is providing teaching to a client who has anemia and a new prescription for epoetin alfa. Which of the following information should the nurse include in the teaching? a) Hospitalization is required when administering each treatment b) The maximum effect of the medication will occur in 6 months c) Hypertension is a common adverse effect of this medication d) Blood transfusions are needed with each treatment

c) HTN is a common adverse effect of epoetin alfa because of the rise in the production of erythrocytes and other blood cell types. Epoetin alfa is a synthetic version of human erythropoietin, Epoetin alfa is used to treat anemia associated with kidney disease or medication therapy. It increases and maintains the rbc level.

The nurse is monitoring a client who had a myocardial infarction. For which of the following complications should the nurse monitor in the first 24 hrs? a) Infective endocarditis b) Pericarditis c) Ventricular dysrhythmias d) Pulmonary emboli

c) ventricular dysrhythmias After an MI, the electrical conduction system of the heart can be irritable and prone to dysrhythmias. Ischemic tissue caused by the infarction can also interfere with the normal conduction patterns of the hearts electrical system.

Which of the following is the formula for mean corpuscular volume (MCV)? a. (Hgb x 10)/RBC b. Hgb/Hct c. (Hct x 10)/RBC d. RBC/Hct

c. (Hct x 10)/RBC

A patient develops unexpected bleeding and the following test results were obtained: -Prolonged PT and APTT -Decreased fibrinogen -Increased fibrin split products -Decreased platelets What is the most probable cause of these results? a. familial afibrinogenemia b. primary fibrinolysis c. DIC d. liver disease

c. DIC

A 56-year-old woman was admitted to the hospital with a history of a moderate to severe bleeding tendency of several years' duration. Epistaxis and menorrhagia were reported. Prolonged APTT was corrected with fresh normal plasma, adsorbed plasma, and aged serum. Deficiency of which of the following is most likely? a. Factor XII b. Factor VIII c. Factor XI d. Factor IX

c. Factor XI

A patient has a normal prothrombin time and a prolonged activated partial thromboplastin time (APTT) using a kaolin activator. The APTT corrects to normal when the incubation time is increased. These results suggest that the patient has: a. hemophilia A (Factor VIII deficiency) b. Hageman factor (XII) deficiency c. Fletcher factor deficiency (prekallikrein) d. Factor V deficiency

c. Fletcher factor deficiency (prekallikrein)

A 15-year-old girl is taking primaquine for a parasitic infection and notices her urine is a brownish color. A CBC shows mild anemia. The laboratorian performing the reticulocyte count notices numerous irregular shaped granules near the periphery of the RBC. These cellular inclusions are most likely: a. Howell-Jolly bodies b. basophilic stippling c. Heinz bodies d. Pappenheimer bodies

c. Heinz bodies

Which one of the following statements concerning vitamin K is NOT true? a. There are two sources of vitamin K: vegetable and bacterial b. Vitamin K converts precursor molecules into functional coagulation factors c. Heparin inhibits the action of vitamin K d. Vitamin K is fat soluble

c. Heparin inhibits the action of vitamin K

A nurse is assessing a client who has atrial fibrillation. Which of the following pulse characteristics should the nurse expect? a. Slow b. Not palpable c. Irregular d. Bounding

c. Irregular With atrial fibrillation, multiple ectopic foci stimulate the atria to contract. The AV node is unable to transmit all of these impulses to the ventricles, resulting in a pattern of highly irregular ventricular contractions and thus an irregular pulse. Atrial fibrillation is an abnormal cardiac rhythm in which the atria are unable to effectively contract because of multiple rapid stimuli causing the atria to depolarize in an organized manner. The atrial rate can range from 300 to 600 bpm, with the ventricular rate being 120 to 200 bpm. The heart's contraction is not normal in the client who has atrial fibrillation. The atria quiver rather than contract, and the ventricles contract in a rapid, chaotic fashion. The ventricular response provides the client with a palpable pulse, although it may be difficult to count the rate. With atrial fibrillation, the amplitude of the client's pulse is highly variable. There is a decrease in ventricular filling, resulting in varying stroke volumes.

The most appropriate screening test for detecting hemoglobin F is: a. osmotic fragility b. dithionite solubility c. Kleihauer-Betke d. heat instability test

c. Kleihauer-Betke

Of the following, the disease most closely associated with pale blue inclusions in granulocytes and giant platelets is: a. Gaucher's disease b. Alder-Reilly anomaly c. May-Hegglin anomaly d. Pelger-Huet anomaly

c. May-Hegglin anomaly

A platelet count done by phase microscopy is 200 x 103/uL (normal, 150-450 x 103/uL). A standardized template bleeding time on the same person is 15 minutes (normal, 4.5 ± 1.5 minutes). This indicates that: a. the Duke method should have been used for the bleeding time b. the manual platelet count is in error c. abnormal platelet function should be suspected d. the results are as expected

c. abnormal platelet function should be suspected

A leukocyte count and differential on a 40-year-old white man revealed: WBC 5.4 x 103/uL Differential Segs 20% Lymphs 58% Monos 20% Eos 2% These data represent: a. absolute lymphocytosis b. relative neutrophilia c. absolute neutropenia d. leukopenia

c. absolute neutropenia

Increased levels of TdT activity are indicative of: a. Burkitt's lymphoma b. acute granulocytic leukemia c. acute lymphocytic leukemia d. eosinophilia

c. acute lymphocytic leukemia

The bone marrow in the terminal stage of erythroleukemia is often indistinguishable from that seen in: a. myeloid metaplasia b. polycythemia vera c. acute myelocytic leukemia d. aplastic anemia

c. acute myelocytic leukemia

The peripheral blood monocyte is an intermediate stage in the formation of the: a. plasmacyte b. Turk irritation cell c. histiocyte d. hairy cell

c. histiocyte

A deficiency of protein C is associated with which of the following? a. prolonged activated partial thromboplastin time (APTT) b. decreased fibrinogen level (less than 100 mg/dL) c. increased risk of thrombosis d. spontaneous hemorrhage

c. increased risk of thrombosis

A term that means varying degrees of leukocytosis with a shift to the left and occasional nucleated red cells in the peripheral blood is: a. polycythemia vera b. erythroleukemia c. leukoerythroblastosis d. megaloblastoid

c. leukoerythroblastosis

Cells that produce antibodies and lymphokines are: a. erythrocytes b. granulocytes c. lymphocytes d. thrombocytes

c. lymphocytes

Which of the following is true of acute lymphoblastic leukemia (ALL)? a. occurs most commonly in children 1-2 years of age b. patient is asymptomatic c. massive accumulation of primitive lymphoid-appearing cells in bone marrow occurs d. children under 1 year of age have a good prognosis

c. massive accumulation of primitive lymphoid-appearing cells in bone marrow occurs

Precursors of tissue macrophages of the reticuloendothelial system most likely are: a. T lymphocytes b. B lymphocytes c. monocytes d. mast cells

c. monocytes

Which of the following is characteristic of platelet disorders? a. deep muscle hemorrhages b. retroperitoneal hemorrhages c. mucous membrane hemorrhages d. severely prolonged clotting times

c. mucous membrane hemorrhages

Dwarf or micromegakaryocytes may be found in the peripheral blood of patients with: a. pernicious anemia b. DIC c. myelofibrosis with myeloid metaplasia d. chronic lymphocytic leukemia

c. myelofibrosis with myeloid metaplasia

Which of the following is associated with pseudo-Pelger-Huet anomaly? a. aplastic anemia b. iron deficiency anemia c. myelogenous leukemia d. Chédiak-Higashi syndrome

c. myelogenous leukemia

Anemia secondary to uremia characteristically is: a. microcytic, hypochromic b. hemolytic c. normocytic, normochromic d. macrocytic

c. normocytic, normochromic

The anemia found in myeloproliferative disorders is usually: a. microcytic, hypochromic b. macrocytic, normochromic c. normocytic, normochromic d. microcytic, normochromic

c. normocytic, normochromic

The stain that identifies intracellular carbohydrate, glycogen, mucopolysaccharide, mucoprotein, glycoprotein, and glycolipid is: a. Sudan black B b. leukocyte alkaline phosphatase (LAP) c. periodic acid-Schiff (PAS) d. peroxidase

c. periodic acid-Schiff (PAS)

In the APTT test, the patient's plasma is mixed with: a. ADP and calcium b. tissue thromboplastin and collagen c. phospholipid and calcium d. tissue thromboplastin and calcium

c. phospholipid and calcium

The ideal capillary blood collection site on a newborn is: a. tip of the thumb b. ear lobe c. plantar surface of the heel d. the great toe

c. plantar surface of the heel

Five preoperative and control samples for APTT had prolonged times when run on an optical density coagulation instrument. The results for patients and controls were confirmed by using an alternate method of mechanical detection. Which of the following would best explain these results? a. incorrect anticoagulant used b. sample carryover c. reagent contamination d. improperly reconstituted controls

c. reagent contamination

Which of the following types of polycythemia is a severely burned patient most likely to have? a. polycythemia vera b. polycythemia, secondary to hypoxia c. relative polycythemia associated with dehydration d. polycythemia associated with renal disease

c. relative polycythemia associated with dehydration

The laboratory tests performed on a patient indicate macrocytosis, anemia, leukopenia, and thrombocytopenia. Which of the following disorders is the patient most likely to have? a. iron deficiency b. hereditary spherocytosis c. vitamin B12 deficiency d. acute hemorrhage

c. vitamin B12 deficiency

Pluripotential Stem Cells

can become many things under (when influenced or affected by) different chemical stimulus

A nurse is assessing a client who has right sided heart failure. Which of the following findings should the nurse expect? a) Decreased capillary refill b) Dyspnea c) Orthopnea d) Dependent edema

d) Dependent edema Blood return from the venous system to the right atrium is impaired by a weakened right heart. The subsequent systemic venous backup leads to development of the dependent edema.

A nurse is assessing a client who has an abdominal aortic aneurysm. Which of the following manifestations should the nurse expect? a) Midsternal chest pain b) Thrill c) Pitting edema in lower extremities d) Lower back discomfort

d) Lower back discomfort Abdominal aortic aneurysm involves a widening, stretching or ballooning of the aorta. Back and abdominal pain indicate that the aneurysm is extending downward and pressing on lumbar spinal nerve roots, causing pain.

Which of the following is the formula for mean corpuscular hemoglobin (MCH)? a. Hct/(RBC x 1000) b. Hgb/Hct c. RBC/Hct d. (Hgb x 10)/RBC

d. (Hgb x 10)/RBC

To prepare 25 mL of 3% acetic acid, how much glacial acetic acid is needed? a. 7.5 mL b. 3.0 mL c. 1.5 mL d. 0.75 mL

d. 0.75 mL

A nurse is caring for a client who is experiencing anaphylactic shock in response to the administration of penicillin. Which of the following medications should the nurse administer first? a. Dobutamine b. Methylprednisone c. Furosemide d. Epinephrine

d. Epinephrine The priority action the nurse should take when using the airway, breathing, circulation approach to client care is to administer epinephrine, a bronchodilator and vasopressor used for allergic reactions to reverse severe manifestations of anaphylactic shock. The nurse should administer dobutamine, a cardiac stimulant used when cardiac decompensation occurs due to anaphylactic shock; however, it is not the first medication the nurse should administer. The nurse should administer methylprednisolone, a corticosteroid, to decrease itching and severe rash; however, it is not the first medication the nurse should administer. The nurse should administer furosemide, a loop diuretic, to improve renal profusion during an anaphylactic crisis; however, it is not the first medication the nurse should administer.

A nurse is caring for a client who is recievig a unit of packed red blood cells. Fifteen minutes following the start of the transfusion, the nurse notes that the client is febrile, with chills and red-tinged urine. Which of the following transfusion reactions should the nurse suspect? a. Febrile b. Allergic c. Acute Pain d. Hemolytic

d. Hemolytic A hemolytic transfusion reaction occurs when antibodies in the recipient's blood react to foreign blood cells introduced by the transfusion. The antibodies bind to the foreign cells and destroy them in a process known as hemolysis. The destroyed cells are excreted by the kidneys (hemoglobinuria), causing the red-tinged urine. Hemolytic transfusion reactions can result in acute renal injury, disseminated intravascular coagulation, and circulatory collapse. A febrile transfusion reaction can occur in clients who have received multiple blood product transfusions. It is a response in which anti-white blood cell (WBC) antibodies react with the WBCs remaining in the blood product. This results in chills, fever, hypotension, tachycardia and tachypnea. Clients who have a history of multiple blood product transfusions may receive leukocyte reduced blood or single-donor HLA matched platelets along with a WBC filter to prevent febrile reactions. Allergic (anaphylactic) transfusion reactions occur most often in clients who have pre-existing allergies. It is thought to be the result of a reaction to the plasma protein contained in the blood product. Manifestations include urticaria, itching, and flushing. In extreme cases, bronchospasm and laryngeal edema, and shock may occur. Onset may occur as late as 24 hr following the transfusion. Clients who have a history of allergies may receive blood products in which the WBCs, plasma, and immunoglobulin A has been removed or the client may be pre-treated with antihistamines and corticosteroids. An acute pain transfusion reaction can occur during or following transfusion with blood products. It manifests as severe chest, joint, and back pain, along with hypertension and flushing of the face and neck. The client is often anxious. Acute pain transfusion reactions are treated symptomatically with medications for pain and rigors.

Which of the following anomalies is an autosomal dominant disorder characterized by irregularly sized inclusions in polymorphonuclear neutrophils, abnormal giant platelets, and often thrombocytopenia? a. Pelger-Huët b. Chédiak-Higashi c. Alder-Reilly d. May-Hegglin

d. May-Hegglin

Thrombocytopenia is a characteristic of: a. classic von Willebrand's disease b. hemophilia A c. Glanzmann's thrombasthenia d. May-Hegglin anomaly

d. May-Hegglin anomaly

A nurse is reviewing the laboratory findings for a client who has idiopathic thrombocytopenia purpura (ITP). which of the following findings should the nurse expect to be decreased? a. WBC b. RBC c. Granulocytes d. Platelets

d. Platelets The nurse should recognize that ITP results from the destruction of platelets by antibodies; therefore, the nurse should expect a platelet level below the expected reference range. The nurse should not expect a decrease in the WBC. The nurse should not expect a decrease in the RBC. The nurse should not expect a decrease in the granulocytes.

A nurse is reviewing the laboratory findings for a client who has idiopathic thrombocytopenia purpura (ITP). which of the following findings should the nurse expect to be decreased? a. WBC b. RBC c. Granulocytes d. Platelets

d. Platelets The nurse should recognize that ITP results from the destruction of platelets by antibodies; therefore, the nurse should expect a platelet level below the expected reference range. -- The nurse should not expect a decrease in the WBC. The nurse should not expect a decrease in the RBC. The nurse should not expect a decrease in the granulocytes.

Which of the following is associated with Glanzmann's thrombasthenia? a. normal bleeding time b. normal ADP aggregation c. abnormal initial wave ristocetin aggregation d. absence of clot retraction

d. absence of clot retraction

Hemolysis in paroxysmal nocturnal hemoglobinuria (PNH) is: a. temperature-dependent b. complement-independent c. antibody-mediated d. caused by a red cell membrane defect

d. caused by a red cell membrane defect

Which of the following are characteristic of polycythemia vera? a. elevated urine erythropoietin levels b. increased oxygen affinity of hemoglobin c. teardrop" poikilocytosis d. decreased or absent bone marrow iron stores

d. decreased or absent bone marrow iron stores

In infectious mononucleosis, lymphocytes tend to be: a. small with little cytoplasm b. normal c. decreased in number d. enlarged and indented by surrounding structures

d. enlarged and indented by surrounding structures

Many microspherocytes and schistocytes and budding off of spherocytes can be seen on peripheral blood smears of patients with: a. hereditary spherocytosis b. disseminated intravascular coagulation (DIC) c. acquired autoimmune hemolytic anemia d. extensive bums

d. extensive bums

A nurse is assessing a client who has left-sided heart failure. which of the following findings should the nurse expect? a. Jugular venous distention b. Abdominal distension c. Dependent edema d. Hacking cough

d. hacking cough A hacking cough is a manifestation of left-sided heart failure that occurs due to pulmonary congestion. Right-sided heart failure raises the pressure and volume within the jugular veins, making them visibly distended when the client is sitting or the head of the bed is elevated more than 30°. Abdominal distension is a manifestation of right-sided heart failure that occurs due to venous congestion. Dependent edema is a manifestation of right-sided heart failure. Pressing edematous skin with a finger leaves a transient indentation (pitting).

A 60-year-old man has a painful right knee and a slightly enlarged spleen. Hematology results include: -Hemoglobin 15 g/dL -Absolute neutrophil count 10.0 x 103/uL -Platelet count 900 x 103/uL -Uncorrected retic count 1% -Normal red cell morphology and indices -A slight increase in bands -Rare metamyeloctye and myelocyte -Giant and bizarre-shaped platelets These results are most compatible with: a. congenital spherocytosis b. rheumatoid arthritis with reactive thrombocytosis c. myelofibrosis d. idiopathic thrombocythemia

d. idiopathic thrombocythemia

Which of the following is typical of polycythemia vera? a. increased serum iron concentration b. decreased thrombocyte count c. increased erythropoietin d. increased leukocyte alkaline phosphatase activity

d. increased leukocyte alkaline phosphatase activity

Which of the following is NOT a characteristic usually associated with hairy cell leukemia? a. pancytopenia b. mononuclear cells with ruffled edges c. splenomegaly d. increased resistance to infection

d. increased resistance to infection

Heparin acts by: a. precipitating calcium b. binding calcium c. activating plasmin d. inhibiting thrombin

d. inhibiting thrombin

Which of the following is NOT a characteristic of hemoglobin H? a. it is a tetramer of beta chains b. it is relatively unstable and thermolabile c. electrophoretically, it represents a "fast" hemoglobin d. its oxygen affinity is lower than that of Hgb A

d. its oxygen affinity is lower than that of Hgb A

Which of the following is most useful in differentiating hemophilias A and B? a. pattern of inheritance b. clinical history c. activated partial thromboplastin time d. mixing studies (substitution studies)

d. mixing studies (substitution studies)

In which of the following disease states are teardrop cells and abnormal platelets most characteristically seen? a. chronic myelocytic leukemia b. multiple myeloma c. thalassemia d. myeloid metaplasia

d. myeloid metaplasia

In synovial fluid, the most characteristic finding in rheumatoid arthritis is: a. cartilage debris b. monosodium urate crystals c. hemosiderin-laden macrophages d. neutrophils with 0.5- to 1.5-um inclusions

d. neutrophils with 0.5- to 1.5-um inclusions

Which of the following platelet responses is most likely associated with hemophilia A (Factor VIII deficiency)? a. defective ADP release; normal response to ADP b. decreased amount of ADP in platelets c. absent aggregation to epinephrine, ADP, and collagen d. normal platelet aggregation

d. normal platelet aggregation

A patient has a tumor that concentrates erythropoietin. He is most likely to have which of the following types of polycythemia? a. polycythemia vera b. polycythemia, secondary to hypoxia c. benign familial polycythemia d. polycythemia associated with renal disease

d. polycythemia associated with renal disease

All of the findings listed below may be seen in acquired hemolytic anemias of the autoimmune variety. The one considered to be the MOST characteristic is: a. increased osmotic fragility b. leukopenia and thrombocytopenia c. peripheral spherocytosis d. positive direct antiglobulin test

d. positive direct antiglobulin test

Which of the following is the most common cause of an abnormality in hemostasis? a. decreased plasma fibrinogen level b. decreased Factor VIII level c. decreased Factor IX level d. quantitative abnormality of platelets

d. quantitative abnormality of platelets

The automated platelet count on an EDTA specimen is 58 x 103/pL. The platelet estimate on the blood smear appears normal, but it was noted that the platelets were surrounding th neutrophils. The next step should be to: a. report the automated platelet count since it is more accurate than a platelet estimate b. warm the EDTA tube and repeat the automated platelet count c. rerun the original specimen since the platelet count and blood smear estimate do not match d. re-collect a specimen for a platelet count using a different anticoagulant

d. re-collect a specimen for a platelet count using a different anticoagulant

An automated platelet count indicates platelet clumping, which is confirmed by examining the smear. The technician should: a. repeat the count on the same sample b. report the automated count c. perform a manual count d. re-collect in sodium citrate

d. re-collect in sodium citrate

On setting up the electronic particle counter in the morning, one of the controls is slightly below the range for the MCV. Which of the following is indicated? a. call for service b. adjust the MCV up slightly c. shut down the instrument d. repeat the control

d. repeat the control

A patient has a high cold agglutinin titer. Automated cell counter results reveal an elevated MCV, MCH, and MCHC. Individual erythrocytes appear normal on a stained smear, but agglutinates are noted. The appropriate course of action would be to: a. perform the RBC, Hgb, and Hct determinations using manual methods b. perform the RBC determination by a manual method; use the automated results for the Hgb and Hct c. repeat the determinations using a microsample of diluted blood d. repeat the determinations using a prewarmed microsample of diluted blood

d. repeat the determinations using a prewarmed microsample of diluted blood

In von Willebrand's disease, platelets give an abnormal aggregation result in the presence of: a. adenosine diphosphate b. epinephrine c. collagen d. ristocetin

d. ristocetin

A characteristic morphologic feature in hemoglobin C disease is: a. macrocytosis b. spherocytosis c. rouleaux formation d. target cells

d. target cells

A useful chemical test for the diagnosis of hairy-cell leukemia is the: a. peroxidase test b. Sudan black B test c. periodic acid-Schiff test d. tartrate-resistant acid phosphatase test

d. tartrate-resistant acid phosphatase test

When evaluating a smear for a reticulocyte count, the technician observes that the red blood cells are overlapping throughout the entire slide. The most likely explanation is: a. grease on the slide prevented even spreading b. improper proportions of blood and stain were used c. the slide was dried too quickly d. the drop used for the slide preparation was too large

d. the drop used for the slide preparation was too large

Thrombocytopenia is....

decrease in platelets

Hypoproteinemia can be caused by a relative proteinemia which is....

decreased concentration due to overhydration

Vacuoles

do not have color inside of them- vacuoles negate the stain

A client is being evaluated in an oncology clinic after referral by the client's provider for suspicion of Hodgkin's disease. Secondary to this possible diagnosis, the nurse should focus on assessing for what? a. bone and joint pain b. enlarged lymph nodes c. difficulty swallowing d. patchy alopecia

enlarged lymph nodes. Hodgkins disease is a malignancy of lymphoid tissue found in the lymph nodes, spleen, liver, and bone marrow.

Hyperchromasia is.....

excessive pigmentation

RBC's

go out into peripheral blood carrying O2 via hemoglobin (protein)

List the characteristics/distribution with Eosinophils:

granulocyte, pink granules, round to oval nucleus, 0-5% in all species.

Granules

have color inside of them

Hypernatremia is...

higher than normal concentrations of sodium in blood. Such elevations are commonly seen in abundant loss of water through gastrointestinal tract along with sodium or low water intake.

Thrombocytosis is....

increase in platelets

What is an aggregate reticulocyte? (Morphologic difference)

it is clumpy w/ribosomes. This indicates a newly released retic. In cats & birds these are the only ones counted.

What is a punctate reticulocyte? (Morphologic difference)

it is polka dotted ribosome. This is an older (more mature) retic, up to weeks in circulation. Under normal conditions cats & birds have punctates.

As part of a primary cancer prevention program, an oncology nurse answers questions from the public at a health fair. When someone asks about laryngeal cancer, the nurse should explain that:

laryngeal cancer is one of the most preventable types of cancer.

A client receiving chemo every few weeks is told by the provider what his nadir was at his last treatment. When asked by the client what nadir is, An appropriate answer by the nurse would be that the nadir is the- a. lowest point your blood count reached after treatment b. highest point your blood count reaches after treatment c. point at which the next dose of chemo can be given again d. point at which chemo is determined to be effective

lowest point your blood count reached after treatment. this occurs approximately 10 days after an antineoplastic treatment.

Reticulocyte count:

made to determine the bone marrows response to anemia. Can patient make its own or does it have to be supplemented?

A client receives a sealed radiation implant to treat cervical cancer. When caring for this client, the nurse should:

maintain the client on complete bed rest with bathroom privileges only.

The total protein test...

measures the total amount of two classes of proteins found in the fluid portion of your blood: albumin and globulin. Proteins are important parts of all cells and tissues. For example, albumin helps prevent fluid from leaking out of blood vessels. Globulins are an important part of your immune system. Is a biochemical test for measuring the total amount of protein in blood plasma or serum.

Changes in what function occur from paraneoplastic syndrome

neurological (movement, sensation, mental function)

The reticulocyte is equivalent to the...

polychromatophilic cell except that it is stained with NMB stain

African-American men are at an increased risk for what type of cancer?

prostate

The corrected WBC count is done to remove what?

remove the # of nucleated RBC's that were in the WBC sample

List the characteristics/distribution with Basophils:

round to oval nucleus, blue to blue gray granules, granulocyte, least common cell in all species

Hemolysis is defined as...

rupturing of RBC's and release of their contents (cytoplasm) into surrounding fluid (EX: blood plasma)

Gauge needle size is...

selected to spare the vein undo damage as well as prevent damage to the blood cells while drawing the blood specimen

Caucasian men are at an increased risk for what type of cancer?

testicular

Explain the refractometer and how it works:

the concentration of ANY solute (dissolved substance) refracts or bends light passing through it to a degree that is proportional to that solutes concentration. Distilled H2O is the index liquid @ 1.000 specific gravity.

Remember this helpful hint...

the degree of illness or disease will alter how drastic of a change there will be to PCV & TP

If you have reactive platelets it is a sign of what?

the platelets are preparing to form a clot

Erythropoiesis is...

the process by which red blood cells (erythrocytes) are produced. It is stimulated by decreased O2 in circulation, which is detected by the kidneys, which then secrete the hormone erythropoietin.[2] This hormone stimulates proliferation and differentiation of red cell precursors, which activates increased erythropoiesis in the hemopoietic tissues, ultimately producing red blood cells.

Sodium levels in animals is important for:

the regulation of blood pressure, blood volume, the transmission of nerve impulses (signals), as well as the maintenance of acid/base balance in the body. The total amount of sodium affects the amount of fluid in blood and around cells. The body continually monitors blood volume and sodium (and other electrolyte) concentrations. When either becomes too high, sensors in the heart, blood vessels, and kidneys detect the increases and stimulate the kidneys to increase sodium excretion, thus returning blood volume to normal. When blood volume or sodium concentration becomes too low, those sensors trigger mechanisms to increase blood volume. These mechanisms include the following: The kidneys stimulate the adrenal glands to secrete the hormone aldosterone. Aldosterone causes the kidneys to retain sodium and to excrete potassium. When sodium is retained, less urine is produced, eventually causing blood volume to increase. The pituitary gland secretes antidiuretic hormone. Antidiuretic hormone causes the kidneys to conserve fluid. Then blood volume increases.

PCV (Packed Cell Volume) is

the volume percentage (%) of RBC's contained in a whole blood sample. It is the ratio of the volume occupied by packed red blood cells to the volume of the whole blood. Centrifuge is used to spin (force) the RBC's to pack together (PCV) and forces the plasma cells out and to the top of the tube. This reading is made on a microhematocrit card reader.

1* Absolute Polycythemia is explained as...

there is NO EPO involved because there is NO HYPOXIA - PCV can be greater than 80%

Hypoproteinemia can be caused by an artifactual proteinemia which is.....

uncalibrating a refractometer, leaving distilled H2O on the refractometer

Indices are...

used to classify anemias. A series of mathematical formulas to determine if anemia is present and the degree to which it is present.

Polychromasia is....

variation of the hemoglobin content of erythrocytes (RBC's)

What is paraneoplastic syndromes

when t-cells in the body attack normal cells instead of cancer cells


Related study sets

Chapter 14 : Warm Up and Flexibility Training

View Set

Maternal Child Nursing Care: Chapter 6-9 Uncomplicated Pregnancy

View Set